Download as docx, pdf, or txt
Download as docx, pdf, or txt
You are on page 1of 49

ANATOMY C) Obturator lnternus

D) Levator ani
1.A 12-year-old boy walks in after falling out of a tree, ANSWER: D
fracturing the upper portion of his humerus. Which of the
following nerves are intimately related to the humerus and 8.
A 31-year-old man was involved in a severe automobile
are most likely to be injured by such a fracture? accident and suffered laceration of the left primary
A) Axillary and musculocutaneous bronchus. The damaged primary bronchus:
B) Radial and ulnar A) Has a larger diameter than the right primary bronchus
C) Radial and axillary B) Often receives more foreign bodies than the right
D) Median and musculocutaneous primary bronchus
ANSWER: C C) Gives rise to the eparterial bronchus
D) Is longer than the right primary bronchus
2.A 14-year-oldboy hits his head on the asphalt road after ANSWER: D
falling off his skateboard. His radiograph reveals damage to
the sella turcica. This is probably due to fracture of which 9.
A 31-year-old patient complains of sensory loss over the
of the following bones? anterior and posterior surfaces of the medial third of the
A) Frontal bone hand and the medial one and one-half fingers. He is
B) Ethmoid bone diagnosed by a physician as having "funny bone"
C) Temporal bone symptoms. Which of the following nerves is injured?
D) Sphenoid bone A) Axillary
ANSWER: D B) Radial
C) Median
3.A 21-year-old woman walks in with a shoulder and arm D) Ulnar
injury after falling during horse-back riding. Examination ANSWER: D
indicates that she cannot adduct her arm because of
paralysis of which of the following muscles? 10. A 32-year-old patient who weighs 275 lb comes to the
A) Teres minor doctor's office. On the surface of the chest, the physician is
B) Supraspinatus able to locate the apex of the heart:
C) Latissimus dorsi A) At the level of the sternal angle
D) Infraspinatus B) In the left fourth intercostal space
ANSWER: C C) In the left fifth intercostal space
D) In the right fifth intercostal space
4.A 22-year-old patient is unable to “unlock” the knee joint ANSWER: C
to permit flexion of the leg. Which of the following
muscles is most likely damaged? 11. A 37-year-old patient with chronic intermittent palpitation
A) Rectus femoris was examined by her physician, and one of the diagnostic
B) Semimembranosus studies included a posterior-anterior chest radiograph.
C) Popliteus Which of the following comprises the largest portion of the
D) Gastrocnemius sternocostal surface of the heart seen on the radiograph?
ANSWER: C A) Left atrium
B) Right atrium
5.A 26-year-old singer visits her physician an ENT surgeon C) Left ventricle
and complains of changes in her voice. A laryngoscopic D) Right ventricle
examination demonstrates a lesion of the superior laryngeal ANSWER: D
nerve, causing weakness of which of the following
muscles? 12. A 38-year-old homebuilder was involved in an accident and
A) Inferior pharyngeal constrictor is unable to supinate his fore- arm. Which of the following
B) Middle pharyngeal constrictor nerves are most likely damaged?
C) Superior pharyngeal constrictor A) Suprascapular and axillary
D) Thyroarytenoid B) Musculocutaneous and median
ANSWER: A C) Axillary and radial
D) Radial and musculocutaneous
6.A 27-year-old woman with a goiter comes to the hospital ANSWER: D
for surgical treatment. The surgeon must ligate the superior
laryngeal artery before surgically resecting the goiter, and 13. A 43-year-old female had been lying down on the hospital
so care must be taken to avoid injury to which of the bed for more than 4 months. Her normal quiet respiration is
following nerves? achieved by contraction of which of the following
A) External laryngeal nerve structures? mastic tissue in the lungs and thoracic wall
B) Internal laryngeal nerve A) Elastic tissue in the lungs and thoracic wall
C) Superior laryngeal nerve B) Serratus posterior superior muscle
D) Hypoglossal nerve C) Pectoralis minor muscle
ANSWER: B D) Serratus anterior muscle
ANSWER: A
7. A 29-year-old woman is admitted to a hospital because the
birth of her child is almost 2 weeks overdue. Tearing of the 14. A 45-year-old woman is suffering from numbness over the
pelvic diaphragm during childbirth leads to paralysis of tip of her nose. Which of the following nerves is most
which of the following muscles? likely to be damaged?
A) Piriformis A) Ophthalmic division of the trigeminal nerve
B) Sphincter urethrae B) Maxillary division of the trigeminal nerve
C) Mandibular division of the trigeminal nerve D) Ureter
D) Facial nerve ANSWER: D
ANSWER: A
22. A 59-year-old woman with pain at the side of her skull
15. A 47-year-old man with a known history of chronic atrial comes to the emergency department. An emergent head CT
fibrillation returns to see his cardiologist for follow-up of scan shows a large lesion in the internal auditory meatus.
his cardiac health. The right atrium is important in this case This condition may progress and damage which of the
because it: following pairs of structures?
A) Receives blood from the oblique cardiac vein A) Vagus and glossopharyngeal nerves
B) Is associated with the apex of the heart B) Internal carotid and vertebral arteries
C) Contains the SA node C) Internal jugular vein and trigeminal nerve
D) Receives the right pulmonary vein D) Facial and vestibulocochlear nerves
ANSWER: C ANSWER: D

16. A 47-year-old woman is unable to invert her foot after she 23. A 60-year-old man is unable to open his eye because of a
stumbled on her driveway. Which of the following nerves rare neuromuscular disease. Which of the following
are most likely injured? muscles would most likely be paralyzed?
A) Superficial and deep peroneal A) Orbicularis oculi
B) Deep peroneal and tibial B) Orbicularis oris
C) Superficial peroneal and tibial C) Frontalis
D) Medial and lateral plantar D) Levator palpebrae superioris
ANSWER: B ANSWER: D

17. A 50-year-old hypertensive woman complains of numbness 24. A 61-year-old woman is found to have ocular lymphoma
and weakness in her left leg and foot. Which of the invading her optic canal. Which of the following structures
following arteries’ occlusion can account for this would most likely be damaged?
complaint? A) Ophthalmic vein
A) Anterior cerebral B) Ophthalmic nerve
B) Anterior choroidal C) Oculomotor nerve
C) Interior carotid D) Ophthalmic artery
D) Middle cerebral ANSWER: D
ANSWER: A
25. A 62-year-old man is incapable of penile erection after
18. A 52-year-old woman slipped and fell and now complains rectal surgery with prostatectomy. The patient most likely
of being unable to extend her leg at the kneejoint. Which of has a lesion of which of the following nerves?
the following muscles was paralyzed as a result of this A) Dorsal nerve of the penis
accident? B) Perineal nerve
A) Semitendinosus C) Hypogastric nerve
B) Sartorius D) Pelvic splanchnic nerve
C) Gracilis ANSWER: D
D) Quadriceps femoris
ANSWER: D 26. A 67-year-old patient has been given a course of antibiotics
by gluteal intramuscular injections after a major abdominal
19. A 56-year-old patient recently suffered a myocardial surgery. To avoid damaging the sciatic nerve during an
infarction of the apex of the heart. The occlusion by injection, the needle should be inserted into which of the
atherosclerosis is in which of the following arteries? following areas?
A) Marginal artery A) Midway between the ischial tuberosity and the
B) Right coronary artery at its origin lesser trochanter
C) Anterior interventricular artery B) Midpoint of the gemelli muscles
D) Posterior interventricular artery C) Superior lateral quadrant of the gluteal region
ANSWER: C D) Inferior medial quadrant of the gluteal region
ANSWER: C
20. A 59-year-old stroke patient is unable to swallow because
of a nerve injury. Which of the following nerves is 27. A bank manager comes in complaining that she cannot flex
unaffected? her proximal interphalangeal joints. Which of the following
A) Hypoglossal nerve muscles appear(s) to be paralyzed on further examination
B) Spinal accessory nerve of her finger?
C) Vagus nerve A) Palmar interossei
D) Facial nerve B) Dorsal interossei
ANSWER: B C) Flexor digitorum profundus
D) Flexor digitorum superficialis
21. A 59-year-old woman comes to a local hospital for uterine ANSWER: D
cancer surgery. A s the uterine artery passes from the
internal iliac artery to the uterus, it crosses superior to 28. A basketball player was hit in the popliteal region by an
which of the following structures that is sometimes opponent's knee. Damage to which of the following arteries
mistakenly ligated during such surgery? might cause ischemia to the extensor muscles of the leg?
A) Ovarian artery A) Popliteal
B) Ovarian ligament B) Deep femoral
C) Uterine tube C) Anterior tibial
D) Posteriortibial C) Biceps femoris
ANSWER: C D) Sartorius
ANSWER: D
29. A capsular stroke is most commonly caused by occlusion of
the following artery/arteries 37. A person suddenly felt the urge to pass out flatus while
A) Anterior cerebral artery inside an elevator filled with passengers. To prevent from
B) Direct branches of the internal carotid artery releasing air, this structure needs to be voluntarily used:
C) Lateral striate arteries A) Levator Ani
D) Posterior communicating artery B) Puborectalis
ANSWER: C C) Internal sphincter
D) External sphincter
30. A congenital diaphragmatic hernia most commonly occurs ANSWER: D
A) on the right anteromedial side
B) on the right posterolateral side 38. A radiologist reviews the chest radiographs of a 27-year-
C) on the left anteromedial side old victim of a car accident. Which of the following
D) on the left posterolateral side structures forms the right border of the cardiovascular
ANSWER: D silhouette?
A) Arch of the aorta
31. A construction worker suffers a destructive injury of the B) Pulmonary trunk
structures related to the anatomic snuffbox. Which of the C) Superior vena cava
following structures would most likely be damaged? D) Ascending aorta
A) Triquetral bone ANSWER: C
B) Trapezoid bone
C) Extensor indicis tendon 39. A rock climber falls on his shoulder; resulting in the lesser
D) Radial artery tubercle chipping off of the humerus. Which of the
ANSWER: D following structures would most likely have structural and
functional damage?
32. A man is unable to hold typing paper between his index and A) Supraspinatus muscle
middle fingers. Which of the following nerves was likely B) lnfraspinatus muscle
injured? C) Subscapularis muscle
A) Radial nerve D) Teres minor muscle
B) Median nerve ANSWER: C
C) Ulnar nerve
D) Musculocutaneous nerve 40. A thoracentesisis performed to aspirate an abnormal
ANSWER: C accumulation of fluid in a 37-year- old patient with pleural
effusion. A needle should be inserted at the midaxillary line
33. A motorcyclist falls from his bike in an accident and gets a be- tween which of the following two ribs to avoid
deep gash that severs the superficial peroneal nerve near its puncturing the lung?
origin. Which of the following muscles is paralyzed? A) Ribs I and 3
A) Fibularis longus B) Ribs 3 and 5
B) Extensor hallucis longus C) Ribs 5 and 7
C) Extensor digitorum longus D) Ribs 7 and 9
D) Fibularis tertius ANSWER: D
ANSWER: A
41. A thoracic surgeon is harvesting a portion of the greater
34. A murder suspect arrives to the emergency department saphenous vein for coronary bypass surgery. He has
(ER) after starting a gunfight with police officers. The observed that this vein runs
suspect was struck by a bullet in the arm, and his median A) Posterior to the medial malleolus
nerve has been damaged. Which of the following symptoms B) In to the popliteal vein
is likely produced by this nerve damage? C) Anterior to the medial condyles of the tibia and
A) Waiter's tip hand femur
B) Claw hand D) Superficial to the fascia lata of the thigh
C) Wrist drop ANSWER: D
D) Ape hand
ANSWER: D 42. A urachal cyst is a remnant of the
A) urogenital sinus
35. A patient has a torn rotator cuff of the shoulder joint as the B) urogenital ridge
result of an automobile accident. Which of the following C) cloaca
muscle tendons is intact and has normal function? D) allantois
A) Supraspinatus ANSWER: D
B) Subscapularis
C) Teres major 43. A21-year-old marine biologist asks about her first bimanual
D) Teres minor examination, and it is explained to her that the normal
ANSWER: C position of the uterus is
A) Anteflexed and anteverted
36. A patient has weakness when flexing both her thigh and B) Retroflexed and anteverted
leg. Which of the following muscles is most likely injured? C) Anteflexed and retroverted
A) Rectus femoris D) Retroverted and retroflexed
B) Semitendinosus ANSWER: A
A) Optic nerve and ophthalmic vein
44. After ingesting a toxic substance found in her friend's B) Ophthalmic vein and ophthalmic nerve
home, a 12-year-old girl is unable to close her lips. Which C) Ophthalmic artery and optic nerve
of the following muscles may be paralyzed? D) Ophthalmic nerve and optic nerve
A) Levator labii superioris ANSWER: C
B) Zygomaticus minor
C) Orbicularis oris 52. During surgery for a malignant parotid tumor in a 69-year-
D) Depressor labii inferioris old woman, the main trunk of the facial nerve is lacerated.
ANSWER: C Which of the following muscles is paralyzed?
A) Masseter muscle
45. An 18-year-old girl is thrust in to the steering wheel after B) Stylopharyngeus muscle
her car hits the vehicle in front of her and stops suddenly. C) Anterior belly of the digastric muscle
She then experiences difficulty in (forced) expiration. D) Buccinator muscle
Which of the following muscles is most likely damaged? ANSWER: D
A) Muscles of the abdominal wall
B) Innermost intercostal muscle 53. During week 2 of development the embryoblast receives its
C) External intercostal muscle nutrients via
D) Diaphragm A) Diffusion
ANSWER: A B) Osmosis
C) Fetal capillaries
46. An 83-year-old man with a previously normal coronary D) yolk sac nourishment
circulation is now having symptoms from an embolism of ANSWER: A
the circumflex branch of the left coronary artery. This
condition would result in ischemia of which of the 54. In which part of the brain is the cerebral aqueduct found?
following areas of the heart? A) Diencephalon
A) Anterior part of the left ventricle B) Mesencephalon
B) Anterior interventricular region C) Metencephalon
C) Posterior interventricular region D) Myelencephalon
D) Posterior part of the left ventricle E) Telencephalon
ANSWER: D ANSWER: B

47. An elderly woman fell at home and fractured the greater 55. In which stage of lung maturation is the blood–air barrier
trochanter of her femur. Which of the following muscles established?
would continue to function normally? A) Embryonic period
A) Piriformis B) Pseudoglandular period
B) Obturator internus C) Canalicular period
C) Gluteus medius D) Terminal sac period
D) Gluteus maximus ANSWER: D
ANSWER: D
56. PNEUMOPERITONEUM detected thru upright abdominal
48. Calot’s Triangle is important because it is the usual location x-ray films is usually found in which of the following
of: potential spaces in the abdomen?
A) Cystic artery A) Subphrenic recess
B) Cystic duct B) Hepatorenal recess
C) Common Hepatic duct C) Bare area of the liver
D) Right Hepatic duct D) Gallbladder fossa
ANSWER: A ANSWER: A

49. Choose the normal quantity of daily CSF production. 57. The anterior fontanel is usually closed by
A) 300 ml A) Birth
B) 400 ml B) age 6 months
C) 500 ml C) age 18 months
D) 600 ml D) age 2 years
ANSWER: C ANSWER: D

50. Coronary angiograms of a 44-year-old man reveal an 58. The cochlear duct contains the spiral organ of Corti and is
occlusion of the circumflex branch of the left coronary derived from which of the following?
artery, which leads to myocardial infarction in which of the A) From both ectoderm and mesoderm
following areas? B) Ectoderm
A) Right and left ventricles C) Endoderm
B) Right and left atria D) Mesoderm
C) Left atrium and ventricle ANSWER: B
D) Apex of the heart
ANSWER: C 59. The cystic artery usually arises from the:
A) Celiac trunk
51. During a game, a 26-year-old baseball player is hit in the B) Left hepatic artery
head by a baseball, which fractures the optic canal. Which C) Common hepatic artery
of the following pairs of structures is most likely to be D) Right hepatic artery
damaged? ANSWER: D
B) anterior spinocerebellar
60. The femur develops from which of the following? C) cuneocerebellar
A) Somite mesoderm D) lateral corticospinal
B) Lateral plate mesoderm ANSWER: A
C) Intermediate mesoderm
D) Extraembryonic mesoderm 69. The surface ectoderm gives rise to which of the following
ANSWER: B structures?
A) Dilator pupillae muscle
61. The indifferent embryo begins phenotypic sexual B) Retina
differentiation during C) Lens
A) week 3 of development D) Sclera
B) week 5 of development ANSWER: C
C) week 7 of development
D) week 12 of development 70. The Y chromosome carries a gene on its short arm that
ANSWER: C codes for
A) testosterone
62. The inferior hemorrhoidal artery that supplies the anal B) MIF
canal distal to the dentate line is a branch of the: C) testes-determining factor (TDF)
A) Sigmoidal artery D) progesterone
B) Left colic artery ANSWER: C
C) Inferior mesenteric artery
D) Internal pudendal artery 71. The “stomach” is derivative of:
ANSWER: D A) Foregut
B) Midgut
63. The most common type of anorectal malformation is C) Hindgut
A) imperforate anus D) ALL OF THESE
B) anal agenesis ANSWER: A
C) anorectal agenesis
D) rectal atresia 72. This viscus is considered half-foregut and half-midgut
ANSWER: C derivative using ampulla of Vater as demarcation:
A) Esophagus
64. The obturator nerve and the sciatic (tibial portion) nerve of B) Stomach
a 15-year-old boy are transected as a result of a motorcycle C) Duodenum
accident. This injury would result in complete paralysis of D) Jejunum
which of the following muscles? ANSWER: C
A) Rectus femoris
B) Biceps femoris, short head 73. TRUE of the gallbladder:
C) Pectineus A) Its wall has no submucosa
D) Adductor magnus B) The tubuloalveolar glands in its lamina propria
ANSWER: D are called Rokitansky- Aschoff sinuses
C) Its main secretory duct is otherwise called duct of
65. The portion of the anal canal distal to the dentate line is Luschka
derived from the: D) All of the above
A) Endoderm ANSWER: A
B) Mesoderm
C) Ectoderm 74. What Colonic flexure separates the transverse from
D) Neuroectoderm descending segment?
ANSWER: C A) Sigmoid flexure
B) Hepatic flexure
66. The pupil in the eye of a 43-year-old patient remains small C) Splenic flexure
even when room lighting is dim. Which of the following D) None of the above
nerves would be injured? ANSWER: C
A) Trochlear nerve
B) Superior cervical ganglion 75. What is the most common congenital malformation of the
C) Oculomotor nerve head and neck region?
D) Ophthalmic nerve A) Anterior cleft palate
ANSWER: B B) Posterior cleft palate
C) Unilateral cleft lip
67. The spinal cord of a newborn baby terminates at: D) Thyrogolossal duct cyst
A) VL1 ANSWER: C
B) VL3
C) VS1 76. What is the predominant serum binding protein that is
D) VS3 solely synthesized in the liver?
E) VS5 A) Albumin
ANSWER: B B) Beta-globulin
C) C-reactive protein
68. The spinal tract involved with the control of trunk muscles D) Transferrin
is: ANSWER: A
A) anterior corticospinal
77. When are the axons of the corticospinal tracts fully 85. Which of the following structures provides the primary
myelinated? support for the cervix of the uterus?
A) In the late embryonic period A) External anal sphincter
B) Mid fetal period B) Broad ligament of the uterus
C) At birth C) Cardinal (transverse cervical) ligament
D) By the end of the second postnatal year D) Round ligament of the uterus
ANSWER: D ANSWER: C

78. Which cranial nerve’s damage result in anosmia? 86. Which of the three primary germ layers forms the
A) I histologically definitive endocardium of the adult heart?
B) II A) Ectoderm
C) III B) Endoderm
D) IV C) Mesoderm
ANSWER: A D) Epiblast
ANSWER: C
79. Which hepatic segment (Couinaud) can be found on the
right side of the liver? 87. Which one of the cranial nerves exits the brainstem from
A) Segment II the posterior aspect?
B) Segment III A) CN I
C) Segment IV B) CN II
D) Segment V C) CN III
ANSWER: D D) CN IV
ANSWER: D
80. Which of the following arteries supplies foregut derivatives
of the digestive system? 88. Which one of the following structures is part of the
A) Celiac Trunk diencephalon?
B) Superior mesenteric artery A) Caudate nucleus
C) Inferior mesenteric artery B) Cerebral hemispheres
D) Right umbilical artery C) Globus pallidus
ANSWER: A D) Thalamus
ANSWER: D
81. Which of the following conditions results from failure of
the anterior neuropore to close? 89. Which process establishes the three definitive germ layers?
A) Hydrocephalus A) Neurulation
B) Anencephaly B) Gastrulation
C) Mongolism C) Craniocaudal folding
D) Craniosynostosis D) Lateral folding
ANSWER: B ANSWER: B

82. Which of the following congenital cardiovascular 90. Which sinus drains the superior surface of the cerebellum?
malformations is most commonly associated with maternal A) Inferior petrosal
rubella infection? B) Inferior sagittal
A) Isolated dextrocardia C) Sigmoid
B) Patent ductus arteriosus D) Straight
C) Patent truncus arteriosus ANSWER: D
D) Coarctation of the aorta
ANSWER: B

83. Which of the following is not a branch of Celiac trunk?


A) Left Gastric Artery
B) Splenic Artery
C) Superior Mesentric Artery
D) Common Hepatic Artery
ANSWER: C

84. Which of the following is TRUE regarding ARTERIAL


supply of the liver and other abdominal organs?
A) The gastroduodenal artery is a branch of the
superior mesenteric artery
B) The superior and inferior mesenteric arteries join
together to form the hepatic artery
C) The cystic artery commonly arises from the
common hepatic artery
D) The celiac trunk gives arises to 3x arteries,
namely: the left gastric, splenic and common
hepatic artery
ANSWER: D
PHYSIOLOGY

1. A 24-year-old woman presents to the emergency


department with severe diarrhea. When she is supine (lying
down), her blood pressure is 90/60 mm Hg (decreased) and
her heart rate is 100 beats/min (increased). When she is
moved to a standing position, her heart rate further
increases to 120 beats/min. Which of the following
accounts for the further increase in heart rate upon
standing?
A) Decreased total peripheral resistance
B) Increased venoconstriction
C) Increased contractility
D) Decreased venous return
ANSWER: D

2. A 27-year-old school teacher “catches” the flu from her


students. Which of the following would occur during the
influenza infection?
A) Phagocytosis of virus by CD4+ T cells
B) Presentation of antigen by CD4+ T cells
C) Killing of virus-infected cells by CD4+ T cells
D) Formation of memory T and B cells
E) Killing of virus-infected cells by neutrophils
ANSWER: D

3. A 32-year-old woman has a positive tuberculin skin test,


helper T cells assist in which of the following ways?
A) Autocrine-mediated inhibition of proliferation of
helper T cells
B) The downregulation of IL-2 receptors on helper T
cells
C) Secretion of interleukins that promote T cell
proliferation
D) Secretion of IL-1
E) Inactivation of macrophages by release of γ-
interferon
ANSWER: C

4. A 35-year-old woman’s physician orders laboratory blood


tests. Her fresh blood is drawn and centrifuged in the
presence of heparin as an anticoagulant to obtain a
hematocrit. The resulting fractions are which of the
following?
A) Serum, packed erythrocytes, and leukocytes
B) Leukocytes, erythrocytes, and serum proteins
C) Plasma, buffy coat, and packed erythrocytes
D) Fibrinogen, platelets, buffy coat, and erythrocytes
E) Albumin, plasma lipoproteins, and erythrocytes
ANSWER: C

5. A 43-year-old woman who has suffered from diabetes for


30 years comes into the clinic. Her hematocrit is 21 and she
has a reduced RBC count. Her serum creatinine is 3.0
(normal 2.0 or below). She has a negative pregnancy test
and is a nonsmoker. Which of the following would best
explain her condition?
A) Decreased hepatic production of erythropoietin
leading to decreased numbers of circulating
reticulocytes in the bloodstream
B) Increased erythropoietin production by the liver
resulting in increased numbers of reticulocytes
C) Decreased renal erythropoietin production
leading to reduced red blood cell production
D) Decreased estrogen levels stimulating hepatic
production of erythropoietin
E) Decreased estrogen levels directly inhibiting red
blood cell production by the bone marrow
ANSWER: C
6. A 62-year-old African-American man presents with 12. A woman runs a marathon in 90°F weather and replaces all
exercise-induced angina. His serum cholesterol is 277 volume lost in sweat by drinking distilled water. After the
mg/dL (normal <200), LDL is 157 (normal <100), HDL is marathon, she will have
43 (normal >35), and triglycerides 170 (normal <150). His A) decreased total body water
BMI is 34 and his coronary risk ratio is 6.84 (normal <5). B) decreased hematocrit
On cardiac catheterization there is occlusion of the left C) decreased intracellular fluid volume
anterior descending and the origin of the right coronary D) decreased plasma osmolarity
artery. The disease process is initiated by which of the ANSWER: D
following?
A) Proliferation of smooth muscle cells 13. According to the sliding filament model, binding sites on
B) Formation of an intimal plaque actin open when
C) Attraction of platelets to collagen microfibrils A) creatine phosphate levels rise
D) Adventitial proliferation B) ATP levels rise
E) Injury to the endothelium C) acetylcholine levels rise
ANSWER: E D) calcium ion levels rise
ANSWER: D
7. A 64-year-old man presents with splenomegaly,
lymphadenopathy, persistent fever, night sweats, and 14. All of the following are phases of gastric secretion and
weight loss. Bone marrow aspiration and biopsy are regulation EXCEPT :
scheduled. Which of the following would be the best place A) Cephalic.
to sample bone marrow? B) Esophageal
A) Iliac crest C) Gasric
B) Sternum D) Intestinal
C) Scapula ANSWER: B
D) Humerus
E) Tibia 15. An increase in arteriolar resistance, without a change in any
ANSWER: A other component of the cardiovascular system, will produce
A) a decrease in total peripheral resistance
8. A lesion of the chorda tympani nerve would most likely B) an increase in capillary filtration
result in C) an increase in arterial pressure
A) impaired olfactory function D) decrease in afterload
B) impaired vestibular function ANSWER: C
C) impaired auditory function
D) impaired taste function 16. An increase in contractility is demonstrated on a Frank-
ANSWER: D Starling diagram by
A) increased cardiac output for a given end-diastolic
9. A newborn girl presents with a mutation in the volume
erythropoietin receptor gene which leads to primary B) increased cardiac output for a given end-systolic
familial erythrocytosis (familial polycythemia). During the volume
5th to 9th months of fetal development, the primary effect C) decreased cardiac output for a given end-diastolic
was on red blood cell production in which of the following? volume
A) Liver D) decreased cardiac output for a given end-systolic
B) Yolk sac volume
C) Spleen ANSWER: A
D) Thymus
E) Bone marrow 17. As a hematologist in the clinic, you diagnose a 34-year-old
ANSWER: E woman with idiopathic thrombocytopenic purpura (ITP).
Which of the following symptoms/characteristics would
10. A police officer in Batangas receives a vaccine for anthrax. you expect in this patient?
The immunological basis for the vaccine is based on which A) Decreased clotting time
of the following facts? B) Normal blood count
A) The primary response has a longer lag period C) Abnormal bruising
B) The secondary response has a shorter duration D) Hypercoagulation
C) The primary response is primarily an IgM E) Light menstrual periods
response ANSWER: C
D) The primary response lacks specificity
E) The primary response generates memory B and T 18. At which site is systolic blood pressure the highest?
cells A) Aorta
ANSWER: E B) Pulmonary artery
C) Right atrium
11. A woman has a plasma osmolarity of 300 mOsm/L and a D) Renal artery
urine osmolarity of 1200 mOsm/L. The correct diagnosis is ANSWER: D
A) syndrome of inappropriate antidiuretic hormone
(SIADH) 19. Bile has two functions, one is to serve as medium for
B) water deprivation excretion of substances and the other is:
C) diabetes insipidus A) To esterify fat
D) drinking large volumes of distilled water B) To emulsify fat
ANSWER: B C) To help in fat oxidation
D) To facilitate bilirubin conjugation A) Microcytic, hypochromatic anemia with smaller
ANSWER: B mature erythrocytes
B) Macrocytic, hyperchromatic anemia with fewer,
20. Blood flow towards which organ remains constant during larger mature erythrocytes
exercise? C) Poikilocytosis and more fragile erythrocytes
A) Heart D) Spherocytosis
B) Skeletal muscles E) No change in erythrocyte size or shape, but a
C) Skin substantial drop in the hematocrit
D) Brain ANSWER: A
ANSWER: D
28. Expression of antigen associated with class I MHC
21. Cardiac output is the product of heart rate and which other molecules is recognized primarily by which of the
parameter? following?
A) Preload A) B cells
B) Afterload B) CD4+ T lymphocytes
C) Total Peripheral Resistance C) CD8+ T lymphocytes
D) Stroke volume D) Plasma cells
ANSWER: D E) Macrophages
ANSWER: C
22. Clonal selection functions to do which of the following?
A) Create the optimal immune response to one 29. Functions of the astrocytes
specific antigen A) Support neuronal metabolism
B) Stimulate immunoglobulin class switching B) Regulate composition of the extracellular fluid
C) Stimulate the production of self-reacting C) Participate in coupling of cerebral blood flow
lymphocytes with neuronal activity
D) Form specific colony-forming units for D) All of the above
erythropoiesis and granulopoiesis in the bone ANSWER: D
marrow
E) Choose the appropriate homing receptors for 30. Gene rearrangement of cytotoxic T cells occurs primarily in
lymphocytes which of the following?
ANSWER: A A) Bone marrow
B) Spleen
23. Compared with a person who ingests 2 L of distilled water, C) Germinal centers
a person with water deprivation will have a D) Thymus
A) higher free-water clearance E) Mesenteric lymph nodes
B) lower plasma osmolarity ANSWER: D
C) lower circulating level of antidiuretic hormone
D) higher rate of H2O reabsorption in the collecting 31. Glucose, the main fuel of the brain, because it is the only
ducts fuel which enters in sufficient amounts to support the
ANSWER: D energy requirements enters the brain and the cells thru:
A) Facilitative transport
24. Compared with the apex of the lung, the base of the lung B) Passive diffusion
has C) Active transport
A) a higher pulmonary capillary PO2 D) Secondary active transport
B) a higher pulmonary capillary PCO2 ANSWER: A
C) a higher ventilation/perfusion (V/Q) ratio
D) the same V/Q ratio 32. Hypoxemia produces hyperventilation by a direct effect on
ANSWER: B the
A) phrenic nerve
25. Compared with the systemic circulation, the pulmonary B) J receptors
circulation has a C) lung stretch receptors
A) higher blood flow D) carotid and aortic body chemoreceptors
B) lower resistance ANSWER: D
C) higher arterial pressure
D) higher capillary pressure 33. If an area of the lung is not ventilated because of bronchial
ANSWER: B obstruction, the pulmonary capillary blood serving that area
will have a PO2 that is
26. Cutting which structure on the left side causes total A) equal to atmospheric PO2
blindness in the left eye? B) equal to mixed venous PO2
A) Optic nerve C) equal to normal systemic arterial PO2
B) Optic chiasm D) higher than inspired PO2
C) Optic tract E) lower than mixed venous PO2
D) Geniculocalcarine tract ANSWER: B
ANSWER: A
34. In relaxed muscle, the myosin-binding site on actin is
27. Erythrocytes may have abnormal shapes and sizes in blocked by
certain diseases. In iron deficiency you would expect to see A) titin
which of the following? B) troponin
C) myoglobin
D) tropomyosin 42. Muscle stretch leads to a direct increase in firing rate of
ANSWER: D which type of nerve?
A) α-Motoneurons
35. In the transport of CO2 from the tissues to the lungs, which B) γ-Motoneurons
of the following occurs in venous blood? C) Group Ia fibers
A) Conversion of CO2 and H2O to H+ and HCO3− D) Group Ib fibers
in the red blood cells ANSWER: C
B) Buffering of H+ by oxyhemoglobin
C) Shifting of HCO3− into the RBCs from plasma in 43. Organs such as the brain and thymus have a more effective
exchange for Cl− blood-barrier because their blood capillaries are of which of
D) Binding of HCO3− to hemoglobin the following types?
ANSWER: A A) Continuous type with few vesicles
B) Fenestrated type with diaphragms
36. In which vascular bed does hypoxia cause C) Fenestrated type without diaphragms
vasoconstriction? D) Discontinuous type with diaphragms
A) Coronary E) Discontinuous type without diaphragms
B) Pulmonary ANSWER: A
C) Cerebral
D) Muscle 44. Pulse pressure is
ANSWER: B A) the highest pressure measured in the arteries
B) the lowest pressure measured in the arteries
37. Increased adrenocorticotropic hormone (ACTH) secretion C) measured only during diastole
would be expected in patients D) determined by stroke volume
A) with chronic adrenocortical insufficiency ANSWER: D
(Addison disease)
B) with primary adrenocortical hyperplasia 45. Selective destruction of the zona glomerulosa of the adrenal
C) who are receiving glucocorticoid for cortex would produce a deficiency of which hormone?
immunosuppression after a renal transplant A) Aldosterone
D) with elevated levels of angiotensin II B) Androstenedione
ANSWER: A C) Cortisol
D) Dehydroepiandrosterone
38. Interleukin 2 is produced by which of the following? ANSWER: A
A) Plasma cells
B) Natural killer cells 46. Tacrolimus (FK506) is prescribed for a patient who
C) CD4+ T lymphocytes received an allogeneic liver transplant. The mechanism of
D) CD8+ T lymphocytes action of tacrolimus is blockage of signal transduction
E) Macrophages pathways in which of the following cells?
ANSWER: C A) T lymphocytes
B) Plasma cells
39. Macrophages are directly involved in immune responses in C) Monocytes
which of the following ways? D) Eosinophils
A) Production of IL-2 E) Mast cells
B) Presentation of antigen ANSWER: A
C) Specific killing of tumor cells
D) Production of antibodies 47. The amygdala is
E) Inactivation of helper T cells A) correlated with fear and anxiety
ANSWER: B B) correlated with learning
C) correlated with pleasure
40. Mechanism used to change how an axon responds to cues D) correlated with mood
in its environment ANSWER: A
A) Increase in cAMP levels and Decrease in cGMP
levels 48. The brain uses what percentage of the body's energy?
B) Local protein synthesis. A) 2%
C) Changes in the composition of receptors in the B) 10%
growth cone. C) 20%
D) All are correct D) 50%
ANSWER: D ANSWER: C

41. Muscle relaxation occurs when 49. The cell membrane of a muscle fiber is called
A) calcium ions are actively transported out of the A) myofibril
sarcoplasmic reticulum B) sarcolemma
B) calcium ions diffuse out of the sarcoplasmic C) sarcoplasm
reticulum D) myofilament
C) calcium ions are actively transported into the ANSWER: B
sarcoplasmic reticulum
D) calcium ions diffuse into the sarcoplasmic 50. The effect of parasympathetic fibers in gastrointestinal tract
reticulum is for:
ANSWER: C A) Vasoconstriction
B) Antiperistalsis
C) Motility &amp; secretion D) Schwann cell, astrocyte, pericytes
D) Pain Sensation ANSWER: A
ANSWER: C
58. The ventricles are completely depolarized during which
51. The greatest pressure decrease in the circulation occurs isoelectric portion of the electrocardiogram (ECG)?
across the arterioles because A) PR interval
A) they have the greatest surface area B) QRS complex
B) they have the greatest cross-sectional area C) QT interval
C) the velocity of blood flow through them is the D) ST segment
highest ANSWER: D
D) they have the greatest resistance
ANSWER: D 59. This protein is the transporter for glucose in the blood brain
barrier:
52. The highest volume of daily secretions in the A) Glut-1
gastrointestinal tract is : B) Glut-2
A) Saliva C) Glut-3
B) Small intestine D) Glut-4
C) Gastric ANSWER: A
D) Bile
ANSWER: B 60. To maintain normal H+ balance, total daily excretion of H+
should equal the daily
53. The hyperpolarized state A) fixed acid production plus fixed acid ingestion
A) occurs at the axon hillock B) HCO3− excretion
B) is due to K ion efflux C) HCO3− filtered load
C) is due to Na ion influx D) titratable acid excretion
D) None of the above E) filtered load of H+
ANSWER: B ANSWER: A

54. The pH of venous blood is only slightly more acidic than 61. True of learning
the pH of arterial blood because A) Long-term potentiation is involved
A) CO2 is a weak base B) Synaptogenesis is required for brain growth
B) there is no carbonic anhydrase in venous blood C) A and B are True
C) the H+ generated from CO2 and H2O is buffered D) None is true
by HCO3– in venous blood ANSWER: C
D) the H+ generated from CO2 and H2O is buffered
by deoxyhemoglobin in venous blood 62. Visual information is analyzed and received by
ANSWER: D A) frontal lobe
B) parietal lobe
55. The shortage of human organs for transplant has focused C) occipital lobe
attention on xenotransplantation as a potential solution for D) temporal lobe
obtaining donor organs. Rejection of a pig pancreas ANSWER: C
transplanted into a human would occur primarily through
which of the following mechanisms? 63. Weakening of the colonic wall due to aging is associated to
A) Preformed antibodies recognize carbohydrates on development of
endothelial cells in the graft. A) Colonic malignancies
B) Tc lymphocytes recognize dendritic cells in the B) Diverticuli
graft C) Hemorrhoids
C) Th lymphocytes recognize macrophages in the D) Irritable bowel syndrome
graft ANSWER: B
D) Plasma cell response to antigens in the β cells of
the islets 64. What is the predominant serum binding protein that is
E) Hyperacute response of T cells to the pancreatic solely synthesized in the liver?
acinar cells A) Albumin
ANSWER: A B) Beta-globulin
C) C-reactive protein
56. The stimuli for secretion of Motilin are the following D) Transferrin
EXCEPT : ANSWER: A
A) Acid
B) Fat 65. When compared with the cones of the retina, the rods
C) Carbohydrate A) are more sensitive to low-intensity light
D) Nerve stimulus B) adapt to darkness before the cones
ANSWER: C C) are most highly concentrated on the fovea
D) are primarily involved in color vision
57. The three cellular elements that compose the blood brain ANSWER: A
barrier
A) Endothelial cells, astrocyte end feet and pericytes 66. Which autonomic receptor mediates an increase in heart
B) Tight junctions, astrocyte end feet, endothelial rate?
cells A) Adrenergic α1 receptors
C) Microglia, oligodendrocyte and astrocyte B) Adrenergic β1 receptors
C) Adrenergic β2 receptors ANSWER: C
D) Cholinergic muscarinic receptors
ANSWER: B 74. Which of the following hormone involved in appetite
regulation is decreased among the elderly?
67. Which gastric cell is properly matched to its secretory A) Insulin
product? B) Leptin
A) Chief cells: Pepsinogen C) Cholecystokinin
B) Goblet cells: intrinsic factor D) PYY
C) Parietal cell: Mucus ANSWER: A
D) Enterochromaffin-like cells: Gastrin
ANSWER: A 75. Which of the following hormones originates in the anterior
pituitary?
68. Which genus of colonic flora is considered symbiotic and A) Dopamine
helps maintain normal GI function and immunity? B) Growth hormone–releasing hormone (GHRH)
A) Clostridium C) Thyroid-stimulating hormone (TSH)
B) Staphylococcus D) Gonadotropin-releasing hormone (GnRH)
C) Escherichia ANSWER: C
D) Bacteroides
ANSWER: D 76. Which of the following inhibits the secretion of growth
hormone by the anterior pituitary?
69. Which hormone is secreted by duodenal cells upon A) Sleep
stimulation by fatty acids and hydrolyzed protein present in B) Stress
the chyme, and is responsible for regulating gastric C) Puberty
emptying and gastric acid secretion as well as pancreatic D) Somatomedins
secretion and gallbladder contraction? ANSWER: D
A) Gastrin
B) Cholecystokinin 77. Which of the following is a cause of metabolic alkalosis?
C) Lipase A) Diarrhea
D) Trypsinogen B) Chronic renal failure
ANSWER: B C) Ethylene glycol ingestion
D) Hyperaldosteronism
70. Which of the following agents or changes has a negative ANSWER: D
inotropic effect on the heart?
A) Increased heart rate 78. Which of the following is a feature of the sympathetic, but
B) Sympathetic stimulation not the parasympathetic nervous system?
C) Norepinephrine A) Preganglionic neurons release norepinephrine
D) Acetylcholine B) Preganglionic neurons release acetylcholine
ANSWER: D C) Preganglionic neurons originate in the
thoracolumbar spinal cord
71. Which of the following BEST describes the regulation of D) Postganglionic neurons synapse on effector
the force of contraction by the cardiac muscle? organs
A) Epinephrine and Norepinephrine activate beta- ANSWER: C
adrenergic receptors causing positive inotropy,
lusitropy and chronotropy.
B) Modulation of extracellular calcium 79. Which of the following is a property of C fibers?
concentration in vivo is an effective way A) Have the slowest conduction velocity of any
regulating the force of contraction. nerve fiber type
C) The primary mechanism responsible for the B) Have the largest diameter of any nerve fiber type
stretch induced increase in the force of C) Are afferent nerves from muscle spindles
contraction is the changes in the overlap of the D) Are afferent nerves from Golgi tendon organs
thick and thin filaments. ANSWER: A
D) Sympathetic nerve stimulation causes a decrease
in the contractile response. 80. Which of the following is an action of parathyroid hormone
ANSWER: A (PTH) on the renal tubule?
A) Stimulation of adenylate cyclase
72. Which of the following causes hyperkalemia? B) Inhibition of distal tubule Ca2+ reabsorption
A) Exercise C) Stimulation of proximal tubule phosphate
B) Alkalosis reabsorption
C) Insulin injection D) Inhibition of production of 1,25-
D) Decreased serum osmolarity dihydroxycholecalciferol
E) Treatment with β-agonists ANSWER: A
ANSWER: A
81. Which of the following is responsible for cardiac
73. Which of the following changes in the stomach is contraction to happen?
attributable to aging? A) Increase in intracellular calcium concentration
A) Increased acid and pepsin secretion B) Increase in intracellular sodium concentration.
B) Shortened gastric emptying time C) Increase in extracellular calcium concentration.
C) Reduced mucus and bicarbonate secretion D) Increase in extracellular sodium concentration
D) Higher gastric prostaglandin E2 levels ANSWER: A
D) Axons from receptor cells synapse in the
82. Which of the following is the site of highest airway prepiriform cortex
resistance? ANSWER: A
A) Largest bronchi
B) Medium-sized bronchi 90. Which of the following statements is TRUE about GI
C) Smallest bronchi physiology?
D) Alveoli A) Gastrointestinal motility is primarily regulated by
ANSWER: B the endocrine system.
B) Enteroendocrine cells are cells located outside of
83. Which of the following is the site of secretion of intrinsic the GI tract that affects GI function.
factor? C) Enterochromaffin cells are endocrine cells within
A) Gastric antrum the GI system not in contact with the lumen of
B) Gastric fundus the GI tract
C) Duodenum D) The autonomic nervous system plays little to no
D) Ileum role in regulation of GI function.
ANSWER: A ANSWER: C

84. Which of the following is the source of energy of the brain 91. Which of the following structures is NOT implicated in
during prolonged states of starvation? impaired traumatic memories?
A) glycogen A) Amygdala
B) glucose B) Pineal Gland
C) palmitate C) Hippocampus
D) acetoacetate D) Thalamus
ANSWER: D ANSWER: B

85. Which of the following is true during inspiration? 92. Which of the following substances has the highest renal
A) Intrapleural pressure is positive clearance?
B) The volume in the lungs is less than the A) Para-amino hippuric acid
functional residual capacity B) Inulin
C) Alveolar pressure equals atmospheric pressure C) Glucose
D) Intrapleural pressure is more negative than it is D) Na+
during expiration E) Cl−
ANSWER: D ANSWER: A

86. Which of the following is TRUE regarding small intestinal 93. Which of the following vasodilator substance accumulates
physiology in aging? in the contracting muscle and is partly responsible in the
A) Overall motility is slower due to apoptosis of initial decrease in vascular resistance in active muscles?
enteric nerve fibers A) Potassium
B) Impaired protein digestion due to decreased B) Calcium
pancreatic secretion of proteases C) Bradykinin
C) Significantly decreased substrate absorption due D) Oxygen radicals
to decreased villi height ANSWER: A
D) Decreased lactase activity may result in lactase
intolerance. 94. Which of the following would cause an increase in both
ANSWER: D glomerular filtration rate (GFR) and renal plasma flow
(RPF)?
87. Which of the following parameters decreases occurs during A) Hyperproteinemia
exercise? B) A ureteral stone
A) Cardiac output C) Dilation of the afferent arteriole
B) Mean arterial pressure D) Dilation of the efferent arteriole
C) Total peripheral resistance ANSWER: C
D) Oxygen consumption
ANSWER: C 95. Which of the following would produce an increase in the
reabsorption of isosmotic fluid in the proximal tubule?
88. Which of the following parts of the body has cortical A) Increased filtration fraction
motoneurons with the largest representation on the primary B) Extracellular fluid volume expansion
motor cortex (area 4)? C) Decreased peritubular capillary protein
A) Shoulder concentration
B) Ankle D) Increased peritubular capillary hydrostatic
C) Fingers pressure
D) Elbow ANSWER: A
ANSWER: C
96. Which statement best describes the changes in changes in
89. Which of the following statements about the olfactory metabolism with aging?
system is true? A) Decreased lean body mass result in lower resting
A) The receptor cells are neurons energy expenditure.
B) The receptor cells are sloughed off and are not B) Decrease in body fat causes decreased leptin
replaced secretion.
C) Axons of cranial nerve (CN) I are A-delta fibers
C)
Dementia and cognitive impairment may cause
hyperactivity among the elderly.
D) Thermogenic effect of food is significantly
increased due increased anabolism
ANSWER: A

97. Which step in steroid hormone biosynthesis is stimulated


by adrenocorticotropic hormone (ACTH)?
A) Cholesterol → pregnenolone
B) Progesterone → 11-deoxycorticosterone
C) 17-Hydroxypregnenolone →
dehydroepiandrosterone
D) Testosterone → estradiol
E) Testosterone → dihydrotestosterone
ANSWER: A

98. Which type of exercise is used to predominately train type


1 muscle fibres?
A) Cardiovascular activities
B) Muscular strength activities
C) Interval training
D) Yoga
ANSWER: A

99. Which volume remains in the lungs after a tidal volume is


expired?
A) Tidal volume
B) Functional residual capacity
C) Expiratory reserve volume
D) Residual volume
ANSWER: B HISTOLOGY

1. A medical student who has chronic lower respiratory


infections seeks the advice of an ear, nose, and throat
specialist. A biopsy of the student’s respiratory epithelium
reveals alterations in certain epithelial structures. This
patient is most likely to have abnormal
A) microvilli
B) desmosomes
C) cilia
D) hemidesmosomes
ANSWER: C

2. ACTH is produced by which of the following cells?


A) Chromophobes in the pars distalis
B) Neurosecretory cells in the median eminence
C) Basophils in the pars distalis
D) Neurons of the paraventricular nucleus in the
hypothalamus
ANSWER: C

3. Characteristics of olfactory epithelium include which one


of the following?
A) It is located in the inferior region of the nasal
cavity.
B) It is classified as simple columnar.
C) It has an underlying lamina propria containing
mucous glands.
D) It has modified cilia, which act as receptors for
odor.
ANSWER: D

4. Clumps of nucleoprotein concentrated near the periphery of


the nucleus are called
A) nuclear pore complex
B) nucleolus
C) heterochromatin
D) outer nuclear membrane
ANSWER: C
D) neurofilaments and microtubules.
5. Contraction in all types of muscle requires calcium ions. ANSWER: B
Which of the following muscle components can bind or
sequester calcium ions? 14. Of the following cell types found in connective tissue,
A) Rough endoplasmic reticulum which is most often present along capillaries and resembles
B) Tropomyosin fibroblasts?
C) Troponin A) Plasma cell
D) Active sites on actin B) Pericyte
ANSWER: C C) Macrophage
D) Mast cell
6. Dense regular connective tissue is present in ANSWER: B
A) capsules of organs
B) basement membrane 15. Prolactin is synthesized and secreted by which of the
C) tendons following cells?
D) skin A) Acidophils in the pars distalis
ANSWER: C B) Basophils in the pars tuberalis
C) Somatotrophs in the pars distalis
7. During the proliferative phase of the menstrual cycle, the D) Basophils in the pars intermedia
functional layer of the endometrium undergoes which of ANSWER: A
the following changes?
A) Blood vessels become ischemic. 16. Spermatozoa are conveyed from the seminiferous tubules to
B) The epithelium is renewed. the rete testis via the
C) The stroma swells because of edema. A) ductus epididymis.
D) Glands become coiled. B) tubuli recti
ANSWER: B C) ductuli efferentes
D) ductus deferens
8. Each smooth muscle cell ANSWER: B
A) has triads associated with its contraction.
B) has dyads associated with its contraction. 17. The basal layer of the uterine endometrium
C) possesses a single central nucleus. A) is sloughed during menstruation
D) is characterized by the absence of sarcolemmal B) has no glands
vesicles. C) is supplied by coiled arteries
ANSWER: C D) is supplied by straight arteries
ANSWER: D
9. Identify the structure that controls movement of proteins in
and out of the nucleus. 18. The blood–brain barrier is thought to exist because
A) Nuclear pore complex capillaries in the central nervous system have which of the
B) Nucleolus following characteristics?
C) Heterochromatin A) Discontinuous basal lamina
D) Outer nuclear membrane B) Fenestrae with diaphragms
ANSWER: A C) Fenestrae without diaphragms
D) A few pinocytic vesicles
10. Intercellular bridges are characteristic of which of the ANSWER: D
following layers of the epidermis?
A) Stratum granulosum 19. The countercurrent multiplier system in the kidney involves
B) Stratum lucidum the exchange of water and ions between the renal
C) Stratum corneum interstitium and
D) Stratum spinosum A) the blood in the vasa recta.
ANSWER: D B) the blood in the peritubular capillary network.
C) the filtrate in the proximal convoluted tubule.
11. Meissner corpuscles are present in which of the following D) the filtrate in the loop of Henle.
regions of the skin? ANSWER: D
A) Dermal reticular layer
B) Dermal papillary layer 20. The endomysium is a connective tissue investment that
C) Hypodermis surrounds
D) Stratum basale A) individual muscle fibers
ANSWER: B B) muscle fascicles
C) individual myofibrils
12. Myelination of peripheral nerves is accomplished by D) an entire muscle
A) astrocytes ANSWER: A
B) oligodendrocytes
C) Schwann cells 21. The epicardium is one of the three layers of the heart. It is
D) neural crest cells A) continuous with the endocardium.
ANSWER: C B) also known as the visceral pericardium.
C) composed of modified cardiac muscle cells.
13. Nissl bodies are composed of D) capable of increasing intraventricular pressure.
A) synaptic vesicles and acetylcholine. ANSWER: B
B) polyribosomes and rough endoplasmic reticulum.
C) lipoprotein and melanin.
22. The histological appearance of a thyroid gland being B) Lymphocyte
stimulated by TSH would show which of the following? C) Platelet
A) Decreased numbers of follicular cells D) Neutrophil
B) Increased numbers of parafollicular cells ANSWER: A
C) Column-shaped follicular cells
D) An abundance of colloid in the lumen of the 31. Which of the following is true for cardiac muscle?
follicle A) T tubules are located at the Z disk.
ANSWER: C B) T tubules have a smaller diameter than those of
skeletal muscle.
23. The site of transcriptional activity in the nucleus is the C) Troponin is absent.
A) nuclear pore complex D) It possesses triads.
B) nucleolus ANSWER: A
C) heterochromatin
D) euchromatin 32. Which of the following is true for mammalian skeletal
ANSWER: D muscle?
A) T tubules are located at the Z disk.
24. The small intestine has three histologically distinct regions. B) T tubules are absent.
Which of the following statements concerning the C) Troponin is absent.
histological differences in the three regions is true? D) It possesses triads.
A) Peyer patches are present only in the ileum. ANSWER: D
B) Goblet cells are present only in the epithelium of
the duodenum. 33. Which of the following is true for smooth muscle?
C) Brunner glands are located in the duodenum and A) T tubules are located at the Z disk.
jejunum but not the ileum. B) It possesses dyads.
D) Lacteals are present only in the lamina propria of C) Caveolae store and release calcium ions.
the ileum. D) It possesses triads.
ANSWER: A ANSWER: C

25. The trachea possesses which one of the following 34. Which of the following materials can be absorbed directly
components? by the surface lining cells of the stomach?
A) Irregular cartilage plates in its wall A) Vitamin B12
B) Skeletal muscle in its wall B) Polysaccharides
C) An epithelium containing only two cell types C) Alcohol
D) A thick basement membrane underlying its D) Triglycerides
epithelium ANSWER: C
ANSWER: D
35. Which of the following possesses specific and azurophilic
26. The type of epithelium associated with the vermilion zone granules?
of the lips is A) Red blood cell
A) stratified squamous nonkeratinized. B) Lymphocyte
B) pseudostratified ciliated columnar. C) Platelet
C) stratified squamous keratinized. D) Neutrophil
D) stratified cuboidal. ANSWER: D
ANSWER: C
36. Which of the following proteins associated with the
27. Which of the following cell types is present in the gastric erythrocyte plasma membrane is responsible for
glands of the pyloric stomach? maintaining the cell’s biconcave disk shape?
A) Goblet cells A) HbA1
B) Mucous neck cells B) HbA2
C) Paneth cells C) Porphyrin
D) Basal cells D) Spectrin
ANSWER: A ANSWER: D

28. Which of the following is an appendage of skin? 37. Which of the following statements about eccrine sweat
A) Meissner corpuscle glands is true?
B) Langerhans cell A) They are absent in thick skin.
C) Krause end-bulb B) They are holocrine glands.
D) Nail C) They have a narrow duct lined by a stratified
ANSWER: D cuboidal epithelium.
D) They secrete an oily material called sebum.
29. Which of the following is derived from myeloblasts? ANSWER: C
A) Basophil
B) Lymphocyte 38. Which of the following statements about hair follicles is
C) Platelet true?
D) Monocyte A) They are always associated with an eccrine sweat
ANSWER: A gland.
B) They are present in thin skin but not in thick skin.
30. Which of the following is derived from reticulocytes? C) Their associated arrector pili muscle is composed
A) Red blood cell of striated fibers.
D) Their hair shaft inserts into the papillary layer of C) They produce keratohyalin granules.
the epidermis. D) hey may give rise to basal cell carcinoma.
ANSWER: B ANSWER: A

39. Which of the following statements concerning adrenal 46. Which of the following statements concerning terminal
parenchymal cells is true? bronchioles is true?
A) Those of the zona fasciculata produce androgens. A) They are part of the conducting portion of the
B) Those of the adrenal medulla produce respiratory system.
epinephrine and norepinephrine. B) They function in gas exchange.
C) Those of the zona glomerulosa produce C) They do not contain ciliated cells.
glucocorticoids. D) They have cartilage plates present in their walls.
D) Those of the cortex contain numerous secretory ANSWER: A
granules.
ANSWER: B 47. Which of the following statements concerning the stratum
granulosum is true?
40. Which of the following statements concerning alveolar A) It contains melanosomes.
macrophages is true? B) It lies superficial to the stratum lucidum.
A) They secrete alpha 1-antitrypsin. C) It is the thickest layer of the epidermis in thick
B) They secrete elastase. skin.
C) They originate from blood neutrophils. D) It contains keratohyalin granules.
D) They may play a role in causing hyaline ANSWER: D
membrane disease.
ANSWER: B 48. Which of the following statements concerning the thymus
is true?
41. Which of the following statements concerning cortical A) Lymphoid nodules form much of the thymic
collecting tubules is always true? cortex.
A) They are lined by a simple epithelium containing B) Epithelial reticular cells form Hassall corpuscles.
two types of cells. C) T cells migrate into the medulla, where they
B) They are also known as the ducts of Bellini. become immunologically competent.
C) They empty on the area cribrosa. D) Most T cells that enter the thymus are killed in
D) They are permeable to water. the medulla.
ANSWER: A ANSWER: B

42. Which of the following statements concerning innervation 49. Which of the following statements concerning the vaginal
of blood vessels is true? mucosa is true?
A) Vasoconstriction is controlled by A) It is lined by stratified columnar epithelium.
parasympathetic nerve fibers. B) It is lined by stratified squamous keratinized
B) Acetylcholine acts directly on smooth muscle epithelium.
cells. C) It possesses no elastic fibers.
C) Acetylcholine acts directly on endothelial cells. D) It is lubricated by glands in the cervix.
D) Vasodilation is controlled by sympathetic nerve ANSWER: D
fibers.
ANSWER: C 50. Which of the following statements concerning thin skin is
true?
43. Which of the following statements concerning respiratory A) It does not contain sweat glands.
bronchioles is true? B) It lacks a stratum corneum.
A) No gas exchange occurs in them. C) It is less abundant than thick skin.
B) They do not have alveoli forming part of their D) It contains hair follicles.
wall. ANSWER: D
C) They contain goblet cells in their lining
epithelium. 51. Which of the following statements concerning triads in
D) Ciliated cells comprise a portion of their lining mammalian skeletal muscle is true?
epithelium. A) They are located in the Z disk.
ANSWER: D B) They consist of two terminal cisternae of the SR
separated by a T tubule.
44. Which of the following statements concerning sebaceous C) They can be observed with the light microscope.
glands is true? D) They are characterized by a T tubule that
A) They do not begin to function until puberty. sequesters calcium ions.
B) They employ the mechanism of holocrine ANSWER: B
secretion.
C) They are present in thick skin. 52. Which of the following statements is characteristic of
D) They secrete only in response to hormones. bone?
ANSWER: B A) Bone matrix contains primarily type II collagen.
B) About 65% of the dry weight of bone is organic.
45. Which of the following statements concerning skin C) Haversian canals are interconnected via
melanocytes is true? Volkmann canals.
A) They synthesize a pigment that protects against D) Bone growth occurs via interstitial growth only.
damage caused by UV radiation. ANSWER: C
B) They are located only in the dermis.
53. Which of the following statements is characteristic of the 61. Which one of the following statements about stratified
perineurium? squamous epithelium is true?
A) It is a fascia surrounding many bundles of nerve A) The surface layer of cells is always keratinized.
fibers. B) The cells in its most superficial layer are
B) It is the fascia surrounding a single nerve fiber. flattened.
C) It is a thin layer of reticular fibers covering C) Its basal cells rest on an elastic lamina.
individual nerve fibers. D) Its cells lack desmosomes.
D) It consists in part of epithelioid cells that ANSWER: B
surround a bundle (fascicle) of nerve fibers.
ANSWER: D 62. Which one of the following statements about the
desmosome is true?
54. Which of the following structures is present in the male A) It is a disk-shaped adhesion site between
urethra but is not present in the female urethra? epithelial cells.
A) Stratified squamous epithelium B) It permits the passage of large proteins from one
B) Transitional epithelium cell to an adjacent cell.
C) Glands of Littre C) It has a plaque made up of many connexons.
D) External sphincter of skeletal muscle D) It facilitates metabolic coupling between adjacent
ANSWER: B cells.
ANSWER: A
55. Which one of the following cells is associated with
antibody production? 63. Which one of the following statements about the gap
A) Basophil junction is true?
B) Lymphocyte A) It extends as a zone around the apical perimeter
C) Platelet of adjacent cells.
D) Monocyte B) It possesses dense plaques composed in part of
ANSWER: B desmoplakins.
C) It permits the passage of ions from one cell to an
56. Which one of the following hormones lowers blood adjacent cell.
calcium levels by inhibiting bone resorption? D) Its adhesion is dependent upon calcium ions.
A) Calcitonin ANSWER: C
B) Epinephrine
C) Parathyroid hormone 64. Which one of the following statements concerning cardiac
D) Prolactin muscle cells is true?
ANSWER: A A) They are spindle shaped.
B) They require an external stimulus to undergo
57. Which one of the following is a glycoprotein across which contraction.
fibroblasts migrate during wound healing? C) They are multinuclear cells.
A) Fibrillin D) They are joined together end to end by
B) Fibronectin intercalated disks.
C) Elastin ANSWER: D
D) Entactin
ANSWER: B 65. Which one of the following statements concerning hyaline
cartilage is correct?
58. Which one of the following is present in the basement A) It is vascular
membrane and is manufactured by connective tissue cells? B) It contains type IV collagen.
A) Fibrillin C) It undergoes appositional growth only.
B) Fibronectin D) It is located at the articular ends of long bones.
C) Elastin ANSWER: D
D) Entactin
ANSWER: B 66. Which one of the following statements is characteristic of
osteocytes?
59. Which one of the following possesses a distinct internal A) They communicate via gap junctions between
elastic lamina? their processes.
A) Capillary B) They contain large amounts of RER.
B) Metarteriole C) They are immature bone cells.
C) Arteriole D) They are housed as isogenous groups in lacunae.
D) Muscular artery ANSWER: A
ANSWER: D
67. Which one of the following statements is correct
60. Which one of the following statements about hyaluronic concerning the periosteum?
acid is true? A) It is devoid of a blood supply.
A) It is a component of elastic fibers B) It produces osteoclasts.
B) It is a glycosaminoglycan. C) It is responsible for interstitial bone growth.
C) It is a proteoglycan with a shape resembling a D) Its inner layer contains osteoprogenitor cells.
bottlebrush ANSWER: D
D) It is sulfated
ANSWER: B 68. Which one of the following statements regarding collagen
is true?
A) It is composed of tropocollagen.
B) Reticular fibers are composed of type II collagen.
C) It is synthesized mostly by mast cells.
D) Elastic fibers are composed of type IV collagen.
ANSWER: A

69. Which one of the following statements regarding


proteoglycans is true?
A) They consist of a core of fibrous protein
covalently bound to glycoproteins. BIOCHEMISTRY
B) They are attached to ribonucleic acid.
C) They are binding sites for deoxyribonucleic acid 1. A 23-year-old female overdosed on acetaminophen in a
(DNA). suicide attempt. She was brought to the emergency room
D) They are composed of a protein core to which (ER) and given a chemical as an emergency treatment. This
glycosaminoglycans are attached. treatment is designed to increase the synthesis of which one
ANSWER: D of the following?
A) NAPQI (N-acetyl-p-benzoquinone imine)
70. Which one of the following structures is located in the renal B) Glutathione
cortex? C) Glycine
A) Vasa recta D) NAPDH
B) Thin limbs of the loops of Henle ANSWER: B
C) Afferent arterioles
D) Interlobar veins 2. A 23-year-old woman is diagnosed with gouty arthritis. Her
ANSWER: C 24-hour uric acid excretion is found to be 1.4 gram.
Reduced activity of which enzyme could plausibly cause
her hyperuricemia?
A) PRPP synthetase
B) Xanthine oxidase
C) Hypoxanthine-guanine phosphoribosyltransferase
D) PRPP aminotransferase
ANSWER: C

3. A 25-year-old female patient underwent a complex surgery


that resulted in damage to the pituitary gland, such that
hypopituitarism resulted. Part of the treatment for this
patient, postsurgery, should be which one of the following?
A) TSH and ACTH should be given orally.
B) Water intake should be restricted to co,pensate
for low vasopressin levels.
C) Thyroxine tablets should be prescribed and taken
regularly by the patient.
D) Cortisol should be administered daily except
during periods of increased stress.
E) Estrogen and progesterone are the only hormones
needed if the patient wishes to remain fertile.
ANSWER: C

4. A 30-year-old male has had multiple episodes of sudden,


severe pain, redness, and swelling of metatarsophalangeal
joint of his great toes. These problems seem to occur after
the man has had a night out on the town with his friends,
when they go barhopping, and the night usually ends with a
cab ride home for the group. This problem would also be
exacerbated if the man eats which one of the following
during his night out?
A) Hamburger
B) Hot dog
C) Chopped liver
D) Chicken wings
ANSWER: C

5. A 32-year-old female presents with 3 months of irregular


menstrual periods and a milky discharge from both breasts.
Under the microscope, the discharge has many fat globules,
but no red blood cells. She is on no medications, has had a
bilateral tubal ligation, and her pregnancy test is negative.
Which ONE of the following would inhibit the release of
the hormone that is being overproduced in this patient?
A) GnRH
B) Dopamine
C) TRH sign of acute alcohol intoxication. A blood test shows an
D) Somatostatin abnormally high ammonia level. The worst treatment for
ANSWER: B this patient would be to give him
A) A lot of good, protein-rich food
6. A 32-year-old male is on a weight maintenance diet, so he B) Benzoic acid or phenylacetic acid
does not want to lose or gain any weight. Which amino acid C) A diet low in proteins but with plenty of vitamins
must be present in the diet so the patient does not go into a D) A broad-spectrum antibiotic to eliminate
negative nitrogen balance? intestinal bacteria
A) Alanine ANSWER: A
B) Arginine
C) Glycine 12. A 65-year-old diabetic patient is found to have greatly
D) Threonine elevated plasma levels of ketone bodies even after a mixed
E) Serine meal. What is the most important regulated enzyme
ANSWER: D determining the rate of ketogenesis?
A) Acyl-CoA dehydrogenase
7. A 35-year-old female presents with central obesity, a B) Pyruvate kinase
“buffalo hump,” Type 2 diabetes, abdominal striae, and C) Adipose tissue triglyceride lipase
“moon facies.” The hormone being overproduced in this D) Acetyl-CoA carboxylase
syndrome is produced from which one of the following ANSWER: C
compounds?
A) Progesterone 13. A Chinese student at a U.S. medical school complains to
B) Testosterone the school physician that he suffers from bouts of flatulence
C) Estradiol and diarrhea shortly after each breakfast. His usual
D) Aldosterone breakfast consists of two candy bars, a small bag of
ANSWER: A peanuts, and three glasses of fresh milk. He never had
digestive problems in his home country, where his diet
8. A 45-year-old female has had three vertebral fractures and consisted only of vegetables, meat, and rice. He has most
received a diagnosis of “early” osteoporosis. She has also likely a low level of
had a number of kidney stones. Routine labs revealed A) Pepsin
hypercalcemia. Increased secretion of a hormone from B) Pancreatic lipase
which gland/organ may lead to this condition? C) Lactase
A) Thyroid D) Trypsin
B) Adrenal ANSWER: C
C) Hypothalamus
D) Parathyroid 14. A college professor, celebrating his 60th birthday, had too
ANSWER: D much foie gras and wine at his celebratory dinner. The next
morning, he awakened with both a hangover and a severe
9. A 45-year-old man developed severe pain in his back, pain in his right great toe, a condition he had experienced a
which, upon going to the emergency department, turned out number of times previously. Being somewhat
to be due to kidney stones. A stone chemical analysis absentminded, the professor then remembered that he had
indicated a buildup of oxalic and glyoxalic acids. These forgotten to take his maintenance medication for this
compounds can accumulate due to a problem in the condition for over 2 weeks. His maintenance medication
metabolism of which one of the following amino acids? most likely blocks which one of the following reactions?
A) Alanine A) IMP to GMP
B) Tryptophan B) Adenosine to inosine
C) Isoleucine C) Hypoxanthine to xanthine
D) Glycine D) dUMP to dTMP
E) Glutamine ANSWER: C
ANSWER: D
15. A common intermediate in the conversion of glycerol and
10. A 52-year-old patient with a round face, acne, and a large lactate to glucose is which one of the following?
hump on the back of his neck complains that he is too weak A) Pyruvate
to mow his lawn. His fasting blood glucose level is 170 B) Oxaloacetate
mg/dL (the reference range is 80 to 100 mg/dL); plasma C) Malate
cortisol level is 62 μg/mL (the reference range is 3 to 31 D) Glucose-6-phosphate
μg/mL); and plasma ACTH level is 0 pg/mL (the reference ANSWER: D
range is 0 to 100 pg/mL). If the patient’s condition is due to
a single cause, the most likely diagnosis is which one of the 16. A deficiency of which one of the following proteolytic
following? enzymes would have the greatest effect on the digestion of
A) Type 1 DM proteins?
B) Type 2 DM A) Trypsin
C) A secretory tumor of the anterior pituitary B) Chymotrypsin
D) A secretory tumor of the posterior pituitary C) Carboxypeptidase A
E) A secretory tumor of the adrenal cortex D) Pepsin
ANSWER: E ANSWER: A

11. A 55-year-old alcoholic is brought to the hospital in a 17. A man has just received his fourth DUI citation. The judge
confused state. The emergency room physician notes that orders an alcohol dependency program complete with a
the patient’s breath has a foul smell, but the patient has no medication that makes him have nausea and vomiting if he
drinks alcohol while taking the medication. The drug- her baby. The best advice to her is which one of the
induced illness is due to the buildup of which one of the following?
following? A) She must consume pure galactose in order to
A) Ethanol produce the galactose moiety of lactose.
B) Acetaldehyde B) She will not be able to breastfeed her baby
C) Acetate because she cannot produce lactose.
D) Acetyl-CoA C) The production of lactose by the mammary gland
ANSWER: B does not require the ingestion of milk or milk
products.
18. A patient is brought to the emergency room after being D) She can produce lactose directly by degrading α-
found by search and rescue teams. He was mountain lactalbumin.
climbing, got caught in a sudden snowstorm, and had to ANSWER: C
survive in a cave. He had no food for 6 days. In adapting to
these conditions, which metabolic process has increased 24. A vegan has been eating low-quality vegetable protein for
rather than decreased? many years and is now exhibiting a negative nitrogen
A) The brain’s use of glucose balance. This may be occurring due to a lack of which one
B) Muscle’s use of ketone bodies of the following in his/her diet?
C) The red blood cells’ use of glucose A) Linoleic acid
D) The brain’s use of ketone bodies B) Starch
ANSWER: D C) Serine
D) Lysine
19. A patient is in septic shock and his tissues are poorly ANSWER: D
perfused and oxygenated. The major end product of glucose
metabolism in these tissues will be an accumulation of 25. A woman suffered through intermittent pain in her stomach
which one of the following? area, particularly after eating. Imaging tests revealed a large
A) Pyruvate gallstone blocking the upper part of the bile duct. Such a
B) Acetyl-CoA blockage would cause an increase in which one of the
C) Lactate following?
D) Urea A) The formation of chylomicrons
E) Citrate B) The recycling of bile salts
ANSWER: C C) The excretion of bile salts
D) The excretion of fat in the feces
20. A patient presents with fatigue, and a blood count reveals a ANSWER: D
macrocytic, hyperchromic anemia. Which one of the
following may account for this type of anemia? 26. After digestion of a piece of cake that contains flour, milk,
A) Lead poisoning and sucrose as its primary ingredients, the major
B) Folate deficiency carbohydrate products entering the blood are which one of
C) Hereditary spherocytosis the following? Choose the one best answer.
D) Sideroblastic anemia A) Glucose, fructose, and galactose
E) Iron deficiency B) Fructose and galactose
ANSWER: B C) Galactose and glucose
D) Fructose and glucose
21. A patient with Type 1 diabetes self-injected insulin prior to ANSWER: A
their evening meal, but then was distracted and forgot to
eat. A few hours later, the individual fainted, and after the 27. All cells in the human body have to maintain a high
paramedics arrived they did a STAT blood glucose level concentration of reduced glutathione to protect themselves
and found it to be 45 mg/dL. The blood glucose level was from oxidative damage. The oxidized form of glutathione is
so low because which one of the following tissues useless for this purpose. What do the cells need to keep
assimilated most of it under these conditions? glutathione in the reduced state?
A) Brain A) NADPH
B) Liver B) NADH
C) Red blood cells C) Cytochrome c
D) Adipose tissue D) Catalase
ANSWER: D ANSWER: A

22. A pharmaceutical company wants to develop an antiobesity 28. An 18-year-old person with type 1 diabetes has not injected
drug that acts directly on adipose tissue metabolism. The her insulin for 2 days. Her blood glucose is currently 600
most promising drug type would be agents that mg/dL (normal values are 80 to 100 mg/dL). Which one of
A) Stimulate β-adrenergic receptors the following cells of her body can still utilize the blood
B) Inhibit adenylate cyclase glucose as an energy source?
C) Stimulate glucose uptake into adipose cells A) Brain cells
D) Inhibit the hormone-sensitive adipose tissue B) Muscle cells
lipase C) Adipose cells
ANSWER: A D) Myorcardium
ANSWER: A
23. A pregnant woman who has a lactase deficiency and cannot
tolerate milk in her diet is concerned that she will not be 29. An alcoholic who went on a weekend binge without eating
able to produce milk of sufficient caloric value to nourish any food was found to have severe hypoglycemia.
Hypoglycemia occurred because the metabolism of ethanol
prevented the production of blood glucose from which one
of the following? Choose the one best answer. 35. Besides the ubiquitous type I collagen, cartilage contains
A) Lactate, glycerol, and oxaloacetate large quantities of
B) Lactate A) Type II collagen and fibronectin
C) Glycerol B) Type VII collagen and elastin
D) Oxaloacetate C) Type II collagen and proteoglycans
ANSWER: A D) Type III collagen and laminin
ANSWER: C
30. An environmentalist attempted to live in a desolate forest
for 6 months, but had to cut his experiment short when he 36. Classic hemophilia is caused by an inherited deficiency of
began to suffer from bleeding gums, some teeth falling out, clotting factor VIII. This deficiency blocks
and red spots on the thighs and legs. This individual is A) The intrinsic pathway of blood clotting
suffering from an inability to properly synthesize which B) The extrinsic pathway of blood clotting
one of the following proteins? C) The final common pathway of blood clotting
A) Myoglobin D) The fibrinolytic system
B) Hemoglobin ANSWER: A
C) Collagen
D) Insulin 37. Consider a normal 25-year-old man, about 70 kg in weight,
ANSWER: C who has been shipwrecked on a desert island, with no food
available, but plenty of freshwater. Which of the following
31. An individual accidentally ingests an overdose of fuel stores is least likely to provide significant calories to
supplements containing chlorogenic acid, a chemical that the man?
inhibits glucose- 6-phosphatase. After an overnight fast, A) Adipose triacylglycerol
this individual, compared with a healthy person, would B) Liver glycogen
exhibit which one of the following? C) Muscle glycogen
A) An increased rate of gluconeogenesis D) Muscle protein
B) An increased rate of glycogenolysis ANSWER: B
C) An increased level of liver glycogen
D) An increased level of blood glucose 38. Cyanide is a potent inhibitor of cell respiration that
ANSWER: C prevents the oxidation of all nutrients. Therefore cyanide
will definitely reduce the cellular concentration of
32. An individual has been determined to have A) Heme groups
hypertriglyceridemia, with a triglyceride level of 350 B) FADH 2
mg/dL (normal is <150 mg/dL). The patient decides to C) CoA
reduce this level by keeping his caloric intake the same, but D) ATP
switching to a low-fat, low-protein, high-carbohydrate diet. ANSWER: D
Three months later, after sticking faithfully to his diet, his
triglyceride level was 375 mg/dL. This increase in lipid 39. Deficiency of which vitamin is most likely to impair the
content is being caused by which component of his new transamination of amino acids?
diet? A) Folic acid
A) Phospholipids B) Thiamine
B) Triglycerides C) Pyridoxine (vitamin B6)
C) Amino acids D) Biotin
D) Carbohydrates ANSWER: C
E) Cholesterol
ANSWER: D 40. During long-term fasting, the liver produces acetyl-CoA by
the β-oxidation of fatty acids. What is the major metabolic
33. An individual has been on a fad diet for 6 weeks, and has fate of this acetyl-CoA?
begun to develop a number of skin rashes, diarrhea, and A) Fatty acid biosynthesis
forgetfulness. These symptoms could have been less severe B) Gluconeogenesis
if the diet contained a high content of which one of the C) Amino acid biosynthesis
following? D) Ketogenesis
A) Tyrosine E) Oxidation in the TCA cycle
B) Tryptophan ANSWER: D
C) Thiamine
D) Thymine 41. Folate antagonists such as methotrexate are used for cancer
ANSWER: B chemotherapy. Which of the following processes in
purine/pyrimidine metabolism is inhibited directly by these
34. An infant who appeared normal at birth began to develop drugs? A coenzyme form of tetrahydrofolate is required for
lethargy, hypothermia, and apnea within 24 hours. An A) De novo synthesis of pyrimidines
analysis of blood components indicated high levels of B) Synthesis of thymine-containing nucleotides from
ammonia and citrulline, and low levels of urea. The most uracil-containing nucleotides
likely defective enzyme in this child is which one of the C) Synthesis of xanthine from purine nucleotides
following? D) Cleavage of the purine ring in uric acid by an
A) Carbamoyl phosphate synthetase I enzyme in the human liver
B) Ornithine transcarbamoylase ANSWER: B
C) Argininosuccinate synthetase
D) Argininosuccinate lyase 42. Glycogen degradation is an important energy source for
ANSWER: C exercising muscle. How many high-energy phosphate
bonds are synthesized by converting one glucose residue in manipulation would enhance glycogen degradation in the
glycogen to lactic acid? liver?
A) 1 A) An inhibitor of α-adrenergic receptors
B) 2 B) An inhibitor of β-adrenergic receptors
C) 3 C) A drug that inhibits the degradation of cAMP
D) 4 D) A drug that activates protein phosphatase-1
ANSWER: C ANSWER: C

43. How can you best describe the function of cytochrome P- 50. Many properties of biological membranes depend on the
450 in xenobiotic metabolism? structure of the lipid bilayer. Typical features of lipid
A) It transfers an electron to adrenodoxin. bilayers include
B) It binds a water molecule, thereby activating it for A) Impermeability for small inorganic ions such as
a nucleophilic attack on the substrate. sodium and protons
C) It accepts two electrons and a proton from B) Rapid exchange of phospholipids between the
NADPH for transfer to the substrate. two leaflets of the bilayer
D) It activates an oxygen molecule. C) High electrical conductivity
E) It acts as an ATPase. D) Lack of lateral mobility of membrane lipids at
ANSWER: D normal body temperature
E) Permeability for proteins
44. In an embryo with a complete deficiency of pyruvate ANSWER: A
kinase, how many net moles of ATP are generated in the
conversion of 1 mole of glucose to 1 mole of pyruvate? 51. Muscle contraction causes an immediate increase in the rate
A) 0 of oxidative phosphorylation because it
B) 1 A) Decreases the pH
C) 2 B) Increases the ADP concentration
D) 3 C) Increases the activity of PFK-1
ANSWER: A D) Decreases the activity of pyruvate dehydrogenase
ANSWER: B
45. In the biosynthetic pathways for the synthesis of heme,
creatine, and guanine, which one of the following amino 52. Mutations in the p53 gene are the most common aberrations
acids directly provides carbon atoms that appear in the final in spontaneous human cancers. The normal p53 protein
product? affects the cell cycle by
A) Serine A) Inducing cell cycle arrest and apoptosis in
B) Glycine response to DNA damage
C) Cysteine B) Inducing the phosphorylation of pRb in response
D) Glutamate to mitogens
ANSWER: B C) Directly inhibiting Cdk inhibitors in response to
cell-cell contact and other growth-inhibiting
46. In the TCA cycle, a role for thiamine pyrophosphate is stimuli
which one of the following? D) Increasing the activity of cyclin-dependent
A) To accept electrons from the oxidation of protein kinases by inducing their phosphorylation
pyruvate and α-ketoglutarate E) Binding and thereby inactivating the products of
B) To accept electrons from the oxidation of many proapoptotic genes
isocitrate ANSWER: A
C) To form a covalent intermediate with the α-
carbon of α-ketoglutarate 53. Patients who had a pancreatectomy (surgical removal of the
D) To form a thioester with the sulfhydryl group of pancreas) should take supplements of digestive enzymes
CoASH with each meal. These enzyme supplements need NOT
ANSWER: C contain
A) α-Amylase
47. Inhibitors of lipoxygenase can be used for the treatment of B) Proteases
asthma because they prevent the formation of C) Lipase
A) Arachidonic acid D) Disaccharidases
B) Leukotrienes ANSWER: D
C) Thromboxanes
D) Prostacyclin 54. Patients with diabetes frequently report changing visual
ANSWER: B acuities when their glucose levels are chronically high.
Which of the following could explain the fluctuating acuity
48. Like many other tissues, the myocardium can use the with high blood glucose levels?
triglycerides in chylomicrons for its own energy needs. The A) Increased sorbitol in the lens
utilization of these triglycerides requires the enzyme B) Decreased fructose in the lens
A) Acetyl-CoA carboxylase C) Increased oxidative phosphorylation in the lens
B) Glucose-6-phosphate dehydrogenase D) Macular degeneration
C) Phospholipase A 2 ANSWER: A
D) Lipoprotein lipase
ANSWER: D 55. Remembering the distribution and solubility of ethanol,
after several drinks with an evening meal, in which of the
49. Liver glycogen is normally synthesized after a meal and following tissues would you find the LEAST amount of
degraded during fasting. What pharmacological alcohol?
A) Brain 62. The generation of metabolic energy from glucose requires a
B) Liver pathway known as glycolysis. What would be the most
C) Fatty tissue appropriate mechanism of regulation for this pathway?
D) Central cornea A) Inhibition of the first irreversible step by glucose
ANSWER: D B) Inhibition of the first irreversible step by ADP
C) Inhibition of the last irreversible step by ADP
56. Some enzymes of the TCA cycle are physiologically D) Inhibition of the first irreversible step by ATP
regulated. The most common regulatory effect on these ANSWER: D
enzymes is
A) Inhibition by ATP 63. The immediate degradation of glycogen under normal
B) Inhibition by ADP conditions gives rise to which one of the following?
C) Inhibition by NAD + A) More glucose than glucose-1-phosphate
D) Stimulation by citrate B) More glucose-1-phosphate than glucose
ANSWER: A C) Equal amounts of glucose and glucose-1-
phosphate
57. Some vitamins have antioxidant properties and therefore D) Neither glucose nor glucose-1-phosphate
are considered promising for the prevention of E) Only glucose-1-phosphate
atherosclerosis, cancer, and age-related degenerative ANSWER: B
diseases. Antioxidant properties have been demonstrated
for all of the following vitamins except 64. The oxygen-binding curve of hemoglobin is sigmoidal
A) Thiamine (vitamin B 1 ) because
B) α-Tocopherol (vitamin E) A) The binding of oxygen to a heme group increases
C) Ascorbic acid (vitamin C) the oxygen affinities of the other heme groups
D) Retinol (vitamin A) B) The heme groups of the α-chains have a higher
ANSWER: A oxygen affinity than do the heme groups of the β-
chains
58. Starting with glucose, the synthesis of which one of the C) The distal histidine allows the hemoglobin
following would require the participation of isocitrate molecule to change its conformation in response
dehydrogenase? to an elevated carbon dioxide concentration
A) Serine D) The subunits are held in place by interchain
B) Alanine disulfide bonds
C) Aspartate ANSWER: A
D) Glutamate
ANSWER: D 65. The parents rush their 3-year-old son to the hospital after
they discover that he has eaten a packet of a “natural”
59. The adrenal cortex contains a sizable collection of enzymes insecticide that they had bought for their garden. According
for steroid hormone synthesis. Two of these enzymes are to the label the active ingredient is rotenone, which inhibits
required for the synthesis of glucocorticoids but not complex I in the respiratory chain. What will be a likely
androgens; therefore, their deficiency leads to an result of this inhibition?
overproduction of adrenal androgens. These two enzymes A) The mitochondria can no longer oxidize succinate
are to fumarate.
A) Aromatase and 17-hydroxylase B) Most or all of the mitochondrial NAD will be in
B) Desmolase and cytochrome P-450 the oxidized state.
C) 11β-Hydroxylase and 21-hydroxylase C) Most or all of the mitochondrial ubiquinone will
D) 17-Hydroxylase and adrenodoxin be in the oxidized state.
ANSWER: C D) The proton gradient across the inner
mitochondrial membrane will be very steep.
60. The brain produces most of its energy by the oxidation of ANSWER: C
glucose; during long-term fasting, however, it can cover
more than half of its energy needs from 66. The pentose phosphate pathway generates which one of the
A) Oxidation of ketone bodies following?
B) Oxidation of its stored glycogen A) NADH, which may be used for fatty acid
C) Oxidation of free fatty acids synthesis.
D) Oxidation of amino acids B) Ribose-5-phosphate, which may be used for the
ANSWER: A biosynthesis of ATP.
C) Pyruvate and fructose 1,6-bisphosphate by the
61. The catecholamines epinephrine and norepinephrine adjust direct action of transaldolase and transketolase.
metabolic activity throughout the body to satisfy the energy D) Xylulose-5-phosphate by one of the oxidative
demands of the working muscles. All of the following reactions.
catecholamine effects are important during physical activity E) Glucose from ribose-5-phosphate and CO2.
except ANSWER: B
A) Stimulation of glycogenolysis in the liver
B) Stimulation of glycogenolysis in skeletal muscle 67. The selective transport of molecules and inorganic ions
C) Inhibition of glycolysis in skeletal muscle across the membrane requires a “gated channel” across the
D) Inhibition of glycolysis in the liver lipid bilayer. The most typical structural feature of these
ANSWER: C gated channels is
A) Several segments of antiparallel β-pleated sheet
structure
B)
Glycolipids forming the inner lining of the 73. Vitamin D can be produced by the action of sunlight on 7-
channel dehydrocholesterol in the skin, but it has to be converted to
C) Lipids that form a covalent bond with the its biologically active form by hydroxylation reactions in
transported solute A) Lungs and brain
D) Several amphipathic α-helices forming the B) Endothelium and intestines
channel C) Skeletal muscle and adrenal cortex
ANSWER: D D) Liver and kidneys
ANSWER: D
68. The synthesis of fatty acids from glucose in the liver is best
described by which one of the following? 74. What is the starting molecule of cholesterol synthesis?
A) The pathway occurs solely in the mitochondria. A) acetyl-CoA
B) It requires a covalently bound derivative of B) acyl-CoA
pantothenic acid. C) glucose
C) It requires NADPH derived solely from the D) pyruvate
pentose phosphate pathway. ANSWER: A
D) The pathway is primarily regulated by isocitrate.
ANSWER: B 75. When an insulin-dependent diabetic patient undergoes
surgery (any surgery), you should monitor his or her blood
69. The transport of glucose across the capillary endothelium glucose level extra carefully because
of cerebral blood vessels (“blood-brain barrier”) is achieved A) Stress hormones antagonize insulin, and the
by facilitated diffusion. This means that patient may therefore require extra insulin
A) Specific inhibition of cerebral glucose uptake is B) Stress reduces insulin release from the pancreas,
not possible a condition known as insulin shock
B) The cerebral glucose uptake is always directly C) Insulin can inhibit the blood clotting system
proportional to the concentration gradient for D) Any physical or psychological stress is likely to
glucose across the endothelium cause hypoglycemia
C) The inhibition of ATP synthesis in the endothelial ANSWER: A
cells will prevent glucose uptake into the brain
D) As long as glucose is only consumed but not 76. When you get up in the morning, 12 hours after dinner,
produced in the brain, the cerebrospinal fluid what is the main source of your blood glucose?
glucose concentration is always less than the A) Dietary glucose
blood glucose concentration B) Liver glycogen
ANSWER: D C) Muscle glycogen
D) Gluconeogenesis from lactate
70. Thiamine deficiency can cause both acute encephalopathy ANSWER: B
and irreversible memory impairment. These problems are
most often seen in thiamine-deficient 77. Which enzyme catalyzes the rate-limiting step for
A) Newborns cholesterol biosynthesis?
B) Persons who abuse alcohol A) Thiolase
C) Diabetics B) HMG-CoA reductase
D) Vegetarians C) HMG-CoA synthase
ANSWER: B D) Cyclase
ANSWER: B
71. Under physiological conditions, the “reversibility” of a
metabolic reaction is affected by all of the following except 78. Which is lipoprotein transports the resynthesized
A) Concentrations of the substrates and products triacyglycerol within the intestinal cells into the lymph?
B) Concentration of the enzyme A) HDL
C) The energy charge if ATP and ADP participate in B) LDL
the reaction C) VLDL
D) The standard free energy change of the reaction D) Chylomicrons
ANSWER: B ANSWER: D

72. Vascular smooth muscle contracts in response to increased 79. Which lipoprotein transports the endogenously synthesized
cytoplasmic calcium. Nevertheless, many natural agents triacylglycerol in the liver into the circulation?
that stimulate the IP 3 /calcium system (including A) HDL
acetylcholine, histamine, and bradykinin) are potent B) LDL
vasodilators. Why? C) VLDL
A) In the vascular smooth muscle cell, calcium D) Chylomicron
rapidly equilibrates across the plasma membrane. ANSWER: C
B) The calcium-calmodulin complex blocks voltage-
gated calcium channels in vascular smooth 80. Which of the following amino acids, considered
muscle. nonessential for most people, is nutritionally essential for
C) Calcium is transferred from endothelial cells to patients with phenylketonuria?
vascular smooth muscle through gap junctions. A) Phenylalanine
D) The calcium-calmodulin complex stimulates NO B) Tyrosine
synthase in endothelial cells. C) Tryptophan
ANSWER: D D) 5-Hydroxytryptophan
ANSWER: B
81. Which of the following characteristics applies to the lipids E)
N-acetylglutamate is a positive allosteric effector
in biological membranes? of ornithine transcarbamoylase.
A) Triglycerides and phosphoglycerides are the most ANSWER: D
abundant lipids in most membranes.
B) Most glycerol-containing lipids are glycolipids. 88. Which one of the following sequences places the
C) Cholesterol is common in the nuclear and inner lipoproteins in the order of most dense to least dense?
mitochondrial membranes but not in the plasma A) HDL/VLDL/chylomicrons/LDL
membrane of most cells. B) HDL/LDL/VLDL/chylomicrons
D) The glycolipids of the plasma membrane are C) LDL/chylomicrons/HDL/VLDL
found in the outer leaflet of the bilayer. D) VLDL/chylomicrons/LDL/HDL
ANSWER: D E) LDL/chylomicrons/VLDL/HDL
ANSWER: B
82. Which one of the following cell types cannot utilize the
TCA cycle or electron transport chain?
A) Brain
B) Red blood cells
C) Liver
D) Kidney
ANSWER: B

83. Which one of the following components of the electron


transport chain only accepts electrons, and does not donate
them?
A) Cytochrome b
B) Oxygen PATHOLOGY
C) Coenzyme Q
D) FMN 1. A 23-year-old man has an anatomical anomaly wherein the
ANSWER: B urethral meatus opens on the ventral side of the penis. This
condition is referred to as?
84. Which one of the following is a property of pyruvate A) Balanitis
dehydrogenase? B) Epispadias
A) The enzyme contains only one polypeptide chain. C) Hypospadias
B) The enzyme requires thiamine pyrophosphate as D) Priapism
a cofactor. ANSWER: C
C) The enzyme produces oxaloacetate from
pyruvate. 2. A 28-year-old man is found to have azoospermia,
D) The enzyme is converted to an active form by bilaterally normal testicular size, and active
phosphorylation. spermatogenesis. Which of the following condition is the
ANSWER: B most likely explanation for patient’s azoospermia?
A) Cryptorchidism
85. Which one of the following is the amino acid in B) Germinal cell aplasia
hemoglobin that accepts H+ and allows hemoglobin to act C) Duct obstruction
as a buffer to acids? D) Klinefelter’s syndrome
A) Alanine ANSWER: C
B) Histidine
C) Serine 3. A 3-year-old boy is brought to the pediatrician because his
D) Threonine mother noticed an abnormal mass in his scrotum while
E) Aspartate changing his diapers. Further workup demonstrates
ANSWER: B elevated levels of serum α-fetoprotein. Which of the
following is the most likely diagnosis?
86. Which one of the following metabolites is used by all cells A) Choriocarcinoma
for glycolysis, glycogen synthesis, and the hexose B) Leydig cell (interstitial) tumor
monophosphate shunt pathway? C) Endodermal sinus tumor
A) Glucose-1-phosphate D) Teratoma
B) Glucose-6-phosphate ANSWER: C
C) UDP-glucose
D) Fructose-6-phosphate 4. A 36-year-old man presents to his primary care physician
E) Phosphoenolpyruvate complaining of painless enlargement of the testis. Further
ANSWER: B laboratory studies reveal an increase in serum hCG. Of the
following, which is the most likely diagnosis?
87. Which one of the following occurs in the urea cycle? A) Dysgerminoma
A) Carbamoyl phosphate is derived directly from B) Seminoma
glutamine and CO2. C) Embryonal carcinoma
B) Ornithine reacts with aspartate to generate D) Teratoma
argininosuccinate. ANSWER: B
C) The α-amino group of arginine forms one of the
nitrogens of urea. 5. A 5-year-old boy is brought to the pediatrician for a
D) Ornithine directly reacts with carbamoyl physical examination prior to beginning elementary school.
phosphate to form citrulline. On examination, the boy has only one palpable testis in the
scrotum. Further examination reveals a palpable mass in the A) Osteocalcin
left inguinal region. This condition is referred to as? B) CD99
A) Cryptorchidism C) CD117
B) orchitis D) CD20
C) Hydrocele ANSWER: B
D) torsion of the spermatic cord
ANSWER: A 11. A 19 year old male with a history of recurrent nasal
epistaxis undergoes resection of a mass located in the
6. A 58-year-old African-American man presents to the posterolateral wall of the roof of the nasal fossa.
emergency department with severe back pain. His history is Histologically, the lesion consists of a bundle of vascular
negative for trauma and he has no other complaints. He structures within a stroma rich in stellate cells and mast
denies urinary frequency, hesitancy, or dysuria. A digital cells. Which of the following hormones is thought to play a
rectal examination confirms the presence of a firm, hard, role in the pathogenesis of this lesion?
asymmetrical, and stony prostate. Imaging of the spine A) Estradiol
suggests osteoblastic involvement of the spine at lumbar B) Serotonin
vertebrae L3 to L4. In addition to an increase in PSA, C) Growth hormone
which serum marker might also be elevated? D) Testosterone
A) Alkaline phosphatase ANSWER: D
B) CA-125
C) Androgens 12. A 24 year old female sought consult for neck pain with
D) α-fetoprotein restriction of neck movement, pain and numbness along the
ANSWER: A medial border of the right scapula and weakness
accompanied by wasting of the right hand. Imaging studies
7. A 10-year-old boy presents with a history of hair loss, show a lobulated marginated soft tissue lesion measuring
deepening of voice, maturation of external genitalia, and 15mm in the right lamina of C4. Histologic examination
aggressive social behavior. Further examination reveals a after resection showed multiple fragmented masses
functioning testicular tumor. What is the best diagnosis? composed of multiple cartilage islands separated by fibro-
A) Gonadoblastoma osseous septa. What is the diagnosis?
B) Seminoma A) Enchondroma
C) Leydig cell tumor B) Chordoma
D) Sertoli cell tumor C) Osteochondroma
ANSWER: C D) Chondrosarcoma
ANSWER: A
8. A 15 year old female with history of recurrent migraine
diagnosed 2 years before the current clinical picture, began 13. A 25 year old patient underwent multiple resections of
having symptoms of non-pulsatile frontal headaches 9 laryngeal papillomas since the age of 16. The lesions
months before hospital admission with worsening appear composed of well differentiated squamous
symptoms in the last 2 months associated with palpitations epithelium with a papillary structure. Mild nuclear atypia
and hypertensive episodes. Which of the following is the with koilocytotic changes are observed. Which of the
most likely diagnosis? following is the most likely etiology of these lesions?
A) Pituitary adenoma A) Exposure to alcohol
B) Renal artery stenosis B) Exposure to cigarette smoking
C) Pheochromocytoma C) Inherited mutations of epidermal growth factor
D) Paraganglioma receptor gene
ANSWER: C D) Viral infection
ANSWER: D
9. A 15-year-male has a bone tumor in the proximal epiphysis
of his right tibia. A biopsy shows the following: high 14. A 25-year-old woman being evaluated for infertility is
cellular density, polygonal cells with well demarcated found to have an abnormal ridge of red, moist granules
borders and thick cell membranes, abundant cytoplasmic located in the upper third of her vagina. Pertinent medical
glycogen, scattered multinucleated giant cells, delicate history is that her mother was treated with diethylstilbestrol
microcalcifications in a chicken-wire pattern, and reticulin (DES) during her pregnancy. A biopsy from the abnormal
fibers around each cell. Which of the following is the most vaginal ridge reveals the presence of benign glands
likely diagnosis? underneath stratified squamous epithelium. Which of the
A) Chondroblastoma following is the most serious long-term complication of this
B) Chondroma abnormality?
C) Chondrosarcoma A) Clear cell carcinoma
D) Chordoma B) Condyloma acuminatum
ANSWER: A C) Extramammary Paget disease
D) Verrucous carcinoma
10. A 17 year old male patient was seen for consult due to an ANSWER: A
enlarging mass on the femoral area. Pelvic Xray showed a
poorly marginated lytic mass with a periosteal reaction or a 15. A 28 year old man was seen for consult due to an enlarging
sunburst pattern. Histology showed broad sheets of small, mass on the zygomatic area resulting to facial deformity.
uniform cells with hyperchromatic nuclei with Gross examination after resection shows intramedullary,
inconspicuous nucleoli, scan cytoplasm and indistinct cell gritty, gray mass expanding the cortex. An impression of
borders. There are also rosette formation and geographic fibrous dysplasia is considered. Which of the following
necrosis seen. Which of the following histologic characteristics is typical of this entity?
immunohistochemical studies would clinch the diagnosis?
A) Spindle cell proliferation with immature woven proliferation of blood vessels arranged in lobules and with
bone arranged in trabeculae that resemble inflammatory infiltrates. What other condition does she
Chinese letters most likely have?
B) Cortically located spindle cell proliferation with A) Dentures causing trauma
osteoblastic rimming B) AIDS
C) Spindle cells in the fibrous stroma that are larger C) Heavy smoking history
and having pleomorphic nuclei with chrmatin D) Pregnancy
clumping ANSWER: D
D) None of the above
ANSWER: A 21. A 35-year-old man presents with a nodular lesion in the
infraauricular region and an enlarged cervical lymph node.
16. A 29 year old male patient presented with a diffuse Lymph node examination shows diffuse eosinophilia with
swelling on the left maxillary region with progressive eosinophilic microabscesses and infiltration of the germinal
enlargement over a period of one year. Excision was done centers. Inaddition, there is prominent vascular hyperplasia
which revealed a biphasic pattern consisting of islands of of postcapillary venules. Based on these findings, what is
cytologically benign hyaline cartilagewith hypercellular the best diagnosis?
areas containing small, primitive-appearing round and A) Angiolymphoid hyperplasia with eosinophilia
spindled mesenchymal cells. What is the diagnosis? B) Castleman’s disease
A) Enchondroma C) Hodgkin lymphoma
B) Chordoma D) Kikuchi disease
C) Osteochondroma E) Kimura’s disease
D) Chondrosarcoma ANSWER: E
ANSWER: D
22. A 38-year-old woman is diagnosed with invasive cervical
17. A 32 year old female presents with constipation, cold squamous cell carcinoma. Which of the following human
intolerance, sluggishness, hypotension and weakness. Neck papilloma virus (HPV) types are least likely to be found in
ultrasound shows bilaterally shrunken thyroid lobes. the tumor?
Thyroidectomy was done. Biopsy showed small thyroid A) 16
follicles with lymphocytic infiltrates and discernible B) 31
germinal follicles. The most common thyroid malignancy C) 18
is: D) 36
A) Papillary thyroid carcinoma ANSWER: D
B) Medullary carcinoma
C) Follicular carcinoma 23. A 39 year old male presents with a mass in the posterior
D) Anaplastic thyroid carcinoma jaw. Imaging shows a multiloculated cyst with a “soap-
ANSWER: A bubble” apppearance in the posterior mandible, which is
associated with an impacted tooth. Biopsy was done
18. A 32 year old male sought consult due to back pain. revealing nests of infiltrating tumor cells with peripheral
Radiographic imaging showed a round, well demarcated, palisading set against a fibrous stroma. Which of the
expansile, radiolucent zone with a peripheral rim of following statements is true regarding this lesion?
sclerosis on the lumbar vertebra. Resection was done which A) Curettage is the treatment of choice to prevent
showed irregular interlacing network of osteoid with fractures
prominent osteoblastic rimming and features of woven B) Recurrence rate is &lt;10% for conventional-type
bone. What is the diagnosis? tumors
A) Osteosarcoma C) The presence of severe cytologic atypia does not
B) Osteoma affect prognosis
C) Osteoblastoma D) Tumors are considered “malignant” only with
D) Paget’s disease documented metastasis
ANSWER: C ANSWER: D

19. A 32year old male sought consult due to decreasing vision 24. A 39-year-old woman presents with severe menorrhagia
of the bilateral aspect of the his view. This was associated and colicky dysmenorrhea. A hysterectomy including
with headaches and irritability. CT scan showed a mass on resection of the fallopian tubes and ovaries is performed.
the base of the brain. Histologic features characteristic of Examination by the pathologist finds a right adnexal cyst
this case include? measuring approximately 2.3 cm in diameter and filled with
A) Composed of uniform, polygonal cells arrayed in clotted blood. Microscopic examination reveals the
sheets or cords with sparse supporting connective presence in the wall of the cyst of endometrial glands,
tissue or reticulin stroma, and hemosiderin pigment. What is the best
B) Consists of nests or cords of stratified squamous diagnosis?
epithelium embedded in a spongy reticulum that A) Adenomyosis
becomes more prominent in the internal layers B) Endometriosis
C) Pseudopallisading of the tumor cells with C) Hydatid cyst
prominent central necrosis D) Hydatidiform mole
D) Hyperchromatic basal cells forming rosettes with ANSWER: B
minimal pleomorphism
ANSWER: A 25. A 42 year old male had an incidental finding of an adrenal
mass thru CT scan for other reasons. The tumor is nodular
20. A 35 year old female sought consult due to a polypoid, and measures 2.5cm in greatest dimension. Biopsy was
anterior maxillary gingival mass that shows a benign done which showed cells similar to a normal adrenal
cortex. The nuclei are small with some pleomorphism. Microscopy revealed small cells with scant cytoplasm, ill-
There are also cells with vacuolated cytoplasm. What is the defined cell borders, finely granular nuclear chromatin and
diagnosis? absent or inconspicuous nucleoli. The cells cells are round,
A) Adrenocortical hyperplasia oval or spindle-shaped and nuclear molding is prominent.
B) Adrenocortical carcinoma Which of the following is true of this tumor?
C) Adrenocortical adenoma A) Prognosis is relatively better compared to other
D) Pheochromocytoma lung tumors
ANSWER: C B) It is the most aggressive of lung tumors
C) It is not associated with smoking
26. A 46-year-old woman undergoes an abdominal D) None of these
hysterectomy for a “fibroid” uterus. The surgeon requests a ANSWER: B
frozen section on the tumor, which is deferred because of
the lesion’s degree of cellularity. Which of the following 32. A 53 year old female was recently diagnosed with well
histologic criteria will be used by the pathologist to differentiated adenocarcinoma. 10 years ago, the patient
determine if this tumor is benign or malignant? underwent colonoscopy with polypectomy. This was signed
A) Mitotic rate out as a neoplastic polyp. Which of the following
B) Cell pleomorphism neoplastic polyps has the greates propensity for malignant
C) Cell necrosis transformation?
D) Tumor size A) Tubular adenoma
ANSWER: A B) Hyperplastic polyp
C) Villous adenoma
27. A 5 year old female was diagnosed with neuroblastoma. D) Tubulovillous adenoma
Which of the following genes may be tested to supportthe ANSWER: C
diagnosis?
A) C-MYC 33. A 56 year old female presents with metastatic carcinoma of
B) L-MYC unknown source. CA-15-3 was requested and was elevated.
C) K-RAS The most likely site of origin of her cancer is?
D) N-MYC A) Breast
ANSWER: D B) Liver
C) Colon
28. A 50 year old female has a gradually enlarging, painless, D) Ovary
right pre-auricular mass. Upon palpation, the mass is firm ANSWER: A
and nontender. Imaging shows a well-circumscribed,
homogenous mass. Biopsy was done and showed 34. A 57 year old female presents with leukocytosis. Lymph
anastomosing strands of stellate and fusiform epithelial node and bone marrow biopsies are performed. Genetic
cells in a myxoid stroma. Which of the following best testing of the tumor demonstrates a t(9; 22). The most
describes this lesion? likely diagnosis is
A) The tumor is a Warthin tumor. A) Acute lymphocytic leukemia
B) The tumor is most often located in the B) Chronic myelocytic leukemia
submandibular gland. C) Burkitt lymphoma
C) It is a highly malignant salivary gland tumor. D) Follicular lymphoma
D) It has a high rate of recurrence after excision. ANSWER: B
ANSWER: D
35. A 60-year-old postmenopausal woman presents with the
29. A 50 year old male was diagnosed to have enchondromas. new onset of uterine bleeding. An endometrial biopsy is
On further examination, there were also spindle cell diagnosed as atypical hyperplasia. Which of the following
hemangiomas noted in his body. This is characteristic of histologic changes is most characteristic of this
what disease? abnormality?
A) Ollier disease A) Crowding of endometrial glands with budding
B) MEN1 and epithelial atypia
C) Mafucci syndrome B) Lymphatic invasion by interlacing bundles of
D) None of the above atypical spindle-shaped cells
ANSWER: C C) Menstrual-type endometrial glands with focal
atypical cystic dilatation
30. A 52 year old non-smoker male underwent biopsy and D) Stromal invasion by malignant glands with focal
showed papillary structures with mucin producing glands areas of atypical squamous differentiation
and cells. The cells are seen invading the interstitium. ANSWER: A
Lymphovascular invasion is seen. Which of the following
is true about this pathology? 36. A 64-year-old male presents with a stomach mass and is
A) Tumors such as this are more likely to have diagnosed with a gastrointestinal stromal tumor (GIST).
EGFR mutations Which of the following protooncogenes is most commonly
B) KRAS mutations are common in this tumor associated with GIST?
C) TP3 are not common in this tumor A) ERB-B1
D) All of the above B) KIT
ANSWER: A C) FMS
D) PDGF-R
31. A 52 year old smoker present with hemoptysis and ANSWER: B
dyspnea. CT scan showed an ill-defined lesion located at
the periphery of the left lung. Biopsy was done.
37. A 65 year old male complains of fatigue, weight loss, C) p21
abdominal pain and hematochezia. Colonoscopy was done D) p16INK4A
and the biopsy revealed an adenocarcinoma. Which of the ANSWER: A
following is a precedent to the condition?
A) Familial adenomatous polyposis 44. All subtypes of Hodgkin lymphoma share the following
B) Peutz-Jeghers polyp features, except
C) Hyperplastic polyp A) Majority of the patients are young adults
D) Tubular adenoma B) Preferential involvement of cervical lymph node
ANSWER: D C) Presence of extensive parenchymal fibrosis
D) Tumor cells surrounded by Tcells
38. A 74 year old male patient presented with cough with E) Scattered mononuclear and multinuclear tumor
expectoration for 2 months. He was a chronic cigarette cells
smoker for 45 years. General physical examination ANSWER: C
revealed clubbing of the nails. No pallor, lymphadenopathy
or organomegaly. Chest radiograph showed a mass lesion 45. All the following features can be seen in the acute phase of
in the mid zone of the left lung. Patient underwent CT HIV lymphadenitis, except:
guided biopsy of the mass lesion. The aspirate yielded A) Aggregates of monocytoid cells along blood
blood mixed sticky material. On microscopy, clusters of vessels
epithelial cells with moderate amount of cytoplasm and a B) Atrophic follicles
round central nucleus background showed abundant C) Diminished mantle zone
chondroid and fibromyxoid elements of uniform spindle D) Numerous tingible body macrophages
cells and numerous fatty spaces. What is the diagnosis? E) Scattered multinucleate giant cells
A) Pulmonary chondrosarcoma ANSWER: B
B) Adenocarcinoma
C) Pulmonary hamartoma 46. All the following findings support a diagnosis of reactive
D) Squamous cell carcinoma lymphoid hyperplasia, except
ANSWER: C A) Distended germinal centers
B) Enlarged, oddly shaped follicles
39. A histologic adjunct in the diagnosis of this case is the C) Frequent mitoses
presence of? D) Germinal centers are bcl-2 positive
A) Orphan annie eye nuclei E) Peripheral rim of inactivated lymphocytes
B) Amyloid deposits ANSWER: D
C) Hurthle cells
D) Capsular invasion 47. Ameloblastoma most commonly occurs in which part of the
ANSWER: C body?
A) Mandible
40. A lung on autopsy shows multiple discrete nodules that are B) Paranasal sinuses
scattered throughout the lobes. Which of the following is C) Maxilla
the most likely etiology? D) Nasopharynx
A) Squamous cell carcinoma ANSWER: A
B) Metastatic tumor
C) Adenocarcinoma 48. An 11 year old patient was diagnosed with an autosomal
D) Neuroendocrine carcinoma dominant syndrome with multiple gastrointestinal
ANSWER: B hamartomatous polyps and mucocutaneous
hyperpigmentation. Genetic studies show STK11 gene
41. A mandibular mass was seen with a histology of curvilinear mutations. Microscopic examination shows an arborizing
trabeculae of woven bone that lack osteoblastic rimming in network of connective tissue, smooth muscle, lamina
a background of fibrous tissue. Which of the following is propria and glands lined by normal appearing intestinal
the genetic mutation associated with this lesion? epithelium. What is the diagnosis?
A) TP53 A) Juvenile polyposis
B) GNAS1 B) Cowden syndrome
C) APC C) Peutz-Jeghers syndrome
D) WT1 D) Familial adenomatous polyposis
ANSWER: B ANSWER: C

42. A patient was underwent thyroidectomy where microscopy 49. An 18 year old female complained of intermittent pain at
revealed fingerlike structures lined by thyrocytes that have the distal femoral area with associated swelling. Xray of the
optically clear nuclei. There were also concentrically distal femur showed an eccentric, metaphyseal, cystic,
lamellated calcific bodies seen. Which of the following is ballooned lytic lesion with cortical destruction. On
the most likely associated genetic mutationin this disease? microscopic examination after excision, showed numerous
A) BRAF cavernous spaces filled blood and lacking an endothelial
B) RET/PTC lining, the spaces are separated by fibrous septa and
C) RAS containing fibroblasts inflammatory cells and giant cells.
D) TRK Which of the following is the diagnosis?
ANSWER: A A) Giant cell tumor of the bone
B) Unicameral bone cyst
43. All of the following are cell cycle inhibitors except C) Telangiectatic Osteosarcoma
A) CDK1 D) Aneurysmal bone cyst
B) p27 ANSWER: D
56. Epidermal growth factor receptor overexpression and/or
50. An 37 year old male patient sought consult due to a slow- amplification is least likely to be encountered with which of
growing mass on the maxillary area with associated the following tumors?
sinusitis and nasal discharge. On Xray, there is a A) Breast carcinoma
radiodense, intramedullary mass with preservation of the B) Leukemia
surrounding structures. Surgery was eventually done. On C) Glioblastoma
gross examination, it showed a nodular, dense cortical D) Ovarian carcinoma
bone. Microscopically, it consists of dense lamellar bone ANSWER: B
with haversian canals without medullary component. What
is the most likely diagnosis? 57. Gastrointestinal stromal tumors are postulated to arise from
A) Osteoma which structure?
B) Osteoblastoma A) Interstitial cells of Cajal
C) Osteoid osteoma B) Gastric columnar epithelium
D) Paget’s disease of the bone C) Gastric parietal cells
ANSWER: A D) Neuroendocrine tissue
ANSWER: A
51. An 81 year old female sought consult due to a painless
parietal mass. Microscopy of the excised mass shows bony 58. Histologic examination of a 3 cm parotid gland tumor
trabeculae that are irregularly thickened and cement lines shows a predominantly solid growth composed of cells
form a mosaic pattern. What is the diagnosis? resembling normal serous cells of the salivary gland. The
A) Osteoma cells have abundant, slightly basophilic, granular cytoplasm
B) Fibrous dysplasia with a centrally located small nucleus. Which of the
C) Osteoid osteoma following is the most likely diagnosis?
D) Paget’s disease of the bone A) Acinic cell carcinoma
ANSWER: D B) Basal cell adenoma
C) Adenoid cystic caricnoma
52. Biopsy from a brain mass showed a hemorrhagic mass D) Mucoepidermoid carcinoma
composed of round to oval cells forming rosettes. The ANSWER: A
centers of these rosettes contain eosinophilic cytoplasmic
material set against a background of thin, fibrovascular, 59. Histology of a distal femoral mass showed bony tissue
anastomosing septa. Diagnosis? capped by hyaline cartilage that contains evenly distributed
A) Pineoblastoma chondrocytes. The nuclei exhibit some atypia and
B) Pineocytoma pleomorphism. The medullary spaces between the
C) Medulloblastoma trabeculae of bone contains adipose tissue and hematopietic
D) Ependymoma tissue. Which of the following is the diagnosis?
ANSWER: B A) Enchondroma
B) Chondroblastoma
53. Biopsy of a lung mass showed tumor cells arranged in a C) Osteochondroma
lepidic pattern. The tumor cells moderate pleomorphism D) Multiple myeloma
with enlarged nuclei and scanty cytoplasm. What is the ANSWER: C
diagnosis?
A) Neuroendocrine carcinoma 60. How is nasal polyposis associated with inflammation?
B) Squamous cell carcinoma A) Prolonged and recurrent inflammation causes
C) Bronchioalveolar carcinoma prolonged stimulation and irritation to the tissues
D) Small cell carcinoma giving rise to metaplasia and hyperplasia
ANSWER: C B) Prolonged and recurrent inflammation causes
sequestration of tissues
54. Biopsy of the lung tumor shows organoid, trabecular, C) Prolonged and recurrent inflammation causes
palisading and rosette arrangements of tumor cells mutation of the APC gene
separated by a delicate fibrovascular stroma. The individual D) Prolonged and recurrent inflammation causes
cells are regular and have uniform nuclei and a moderate atrophic changes to the epithelium
amount of eosinophilic cytoplasm. What is the diagnosis? ANSWER: A
A) Squamous cell carcinoma
B) Neuroendocrine carcinoma 61. In addition to T-lymphocytes, which of the following cells
C) Adenocarcinoma populate the paracortical region of the lymph node?
D) Small cell carcinoma A) B-lymphocytes
ANSWER: B B) Dendritic reticulum cells
C) Interdigitating cells
55. Common hormones and hormone-like factors are D) Mast cells
elaborated in paraneoplastic syndrome include the E) Plasma cells
following except ANSWER: C
A) Growth hormone
B) Antidiuretic hormone 62. In the adenoma-carcinoma sequence in the development of
C) Gonadotropins adenocarcinoma, the “first hit” pertains to a mutation of
D) Parathormone which gene?
ANSWER: A A) RAS
B) p53
C) Rb gene
D) APC
ANSWER: D C) Colon cancer
D) Melanoma
63. Inherited mutations in the APC/beta-catenin tumor ANSWER: A
suppression gene are associated with which of the
following? 70. The gene fusion associated with Ewing’s sarcoma is?
A) Familial adenomatous polyposis A) EWS-FLI1
B) Melanoma B) Bcl-abr
C) Osteosarcoma C) SSH-APC
D) Ovarian carcinoma D) WT1-FLI1
ANSWER: A ANSWER: A

64. Invasion of the extracellular matrix by tumor cells is 71. The immunohistologic marker for the most common type
important in tumor infiltration and in gaining access to of Non-Hodgkin lymphoma affecting the nasopharynx is?
lymphatic and vascular spaces. All of the following are A) Cytokeratin
important steps in the process of tumor invasion of B) CD3
extracellular matrix EXCEPT C) CD20
A) Attachment to matrix components D) Synaptophysin
B) Detachment of tumor cells from each other ANSWER: C
C) Migration of tumor cells
D) Reaggregation of tumor cells 72. The majority of endometrial polyps have which of the
ANSWER: D following features?
A) Contain dilated thick-walled blood vessels
65. Lung biopsy from a 45 year old female shows sheets of B) Decidual changes are frequent
large, polygonal cells with macronuclei and prominent 1-2 C) Do not contain hyperplastic glands
nucleoli and intercellular bridges. Which of the following is D) Often show foci of adenocarcinoma
true of this tumor? ANSWER: A
A) The cytoplasm of the tumor cells contain
intracytoplasmic mucin 73. The most common malignant tumor of the vagina is
B) Squamous metaplasia and dysplasia precedes this strongly associated with HPV infection, and its most
tumor important risk factor is previous cervical carcinoma. Which
C) TP53 mutations are uncommon of the following is it?
D) All of the above A) Clear cell adenocarcinoma
ANSWER: B B) Embryonal rhabdomyosarcoma
C) Endometrioid adenocarcinoma
66. Progression through the S phase of the cell cycle and the D) Squamous cell carcinoma
initiation of DNA replication involves the formation of ANSWER: D
which complex?
A) Cyclin D–CDK4 74. The most common site of distant metastasis of colonic
B) Cyclin E–CDK2 adenocarcinoma is which organ?
C) Cyclin A–CDK4 A) Lungs
D) Cyclin A–CDK2 B) Liver
ANSWER: B C) Brain
D) Kidneys
67. Progressive transformation of germinal centers is ANSWER: B
associated with all the following findings, except
A) Occurs in young adults 75. The most typical xray bone change due to multiple
B) Occasional Reed-Sternberg cells are present in myeloma is:
the interfollicular space A) Codman triangle
C) Presence of a large lymphoid nodule that is 3–5 B) Expansion of bone and thickening of cortex
times the diameter of an adjacent follicle C) Onionskin appearance
D) Small lymphocytes infiltrate the germinal centers D) Punched out osteolytic lesions
E) Usually presents as a single, enlarged lymph node ANSWER: D
ANSWER: B
76. The occurrence of Nasopharyngeal carcinoma is strongly
68. Prolonged unopposed estrogen stimulation in an adult associated with which infection?
woman increases the risk of development of endometrial A) Epstein Barr virus
hyperplasia and subsequent carcinoma. Which of the B) Human Papillomavirus
following is the most common histologic appearance for C) Human Immunodeficiency virus
this type of cancer? D) Cytomegalovirus
A) Adenocarcinoma ANSWER: A
B) Clear cell carcinoma
C) Small cell carcinoma 77. The protracted course of Barrett’s esophagus may lead to
D) Squamous cell carcinoma the occurrence of which malignancy?
ANSWER: A A) Squamous cell carcinoma
B) Neuroendocrine carcinoma
69. Testing for protooncogene H-RAS point mutations is most C) Adenocarcinoma
likely to be positive in which of the following cancers? D) Gastrointestinal stromal tumor
A) Bladder cancer ANSWER: C
B) Lung cancer
78. The two-hit hypothesis of oncogenesis proposed by 85. Which of the following carcinomas is least likely to
Knudson explains the inherited and sporadic occurrence of metastasize to the supraclavicular lymph nodes?
which of the following tumors? A) Colon
A) Meningioma B) Esophagus
B) Retinoblastoma C) Lung
C) Neuroblastoma D) Pancreas
D) Wilms tumor E) Stomach
ANSWER: B ANSWER: A

79. This is an uncommon, highly malignant vaginal tumor in 86. Which of the following conditions is not a cause of
infants and children consisting of tumor cells that are small, generalized lymphadenopathy?
with oval nuclei and small eccentric cytoplasmic A) Autoimmune hemolytic anemia
protrusions. Grossly this tumor is polypoid, bulky masses B) Hyperthyroidism
composed of grapelike clusters that can protrude from the C) Metastatic carcinoma
vagina. D) Lymphoma
A) Condyloma acuminate E) Sarcoidosis
B) Hidradenoma papilliferum ANSWER: C
C) Extra mammary paget disease
D) Sarcoma botyroides 87. Which of the following features is most characteristic of
ANSWER: D hyperplastic colorectal polyps?
A) Decreased mucin content
80. Tumors may stimulate angiogenesis. All of the following B) Inconspicuous basement membrane
are reasons why a tumor would promote angiogenesis C) Increased mitosis at the base of the glands
except D) Mild to moderate cellular atypia
A) It facilitates metastasis ANSWER: C
B) It helps supply oxygen
C) It helps supply nutrients 88. Which of the following histomorphic features is present in
D) It promotes apoptosis Pleomorphic adenoma?
ANSWER: D A) Chondromyxoid stroma
B) Spindle cells
81. Warthin-Finkeldey type of giant cells are least likely to be C) Squamous metaplasia
associated with which of the following conditions? D) Eosinophilic cytoplasm
A) Follicular lymphoma ANSWER: A
B) Gonococcal lymphadenitis
C) HIV lymphadenitis 89. Which of the following malignant salivary gland tumors is
D) Lymphocyte-predominant Hodgkin lymphoma strongly associated with perineural invasion secondary to
E) Measles lymphadenitis the release of brain neurotropic peptide?
ANSWER: B A) Mucoepidermoid carcinoma
B) Malignant mixed tumor
82. What is the reason behind a strong association of smoking C) Acinic cell carcinoma
and squamous cell carcinoma of the larynx? D) Adenoid cystic carcinoma
A) Smoking creates stress to the lining epithelium of ANSWER: D
the larynx and causes columnar metaplasia
B) Smoking induces squamous metaplasia to the 90. Which of the following oncogenesis most frequently
lining epithelium rearranged in follicular lymphoma?
C) Smoking induces activation of the p16 gene A) bcl-2
D) Smoking induces overexpression of the p53 gene B) bcl-6
ANSWER: B C) myc
D) p16
83. Which HPV proteins is involved with facilitating p53 E) p53
degradation, resulting in a block in apoptosis and decreased ANSWER: A
p21 cell cycle inhibitor activity?
A) E1 91. Which of the following risk factors is least likely to cause
B) E5 bladder cancer?
C) E3 A) Alcohol
D) E7 B) Aromatic amines
ANSWER: D C) Aniline dyes
D) Cigarette smoking
84. Which of the following applies to uterine ANSWER: A
leiomyosarcomas?
A) Arise from malignant transformation of 92. Which of the following special stains will aid in the
leiomyomas diagnosis of Mucoepidermoid carcinoma?
B) Consist usually of clear cells with sharp outlines A) Lugol’s Iodine
C) Differentiation from leiomyomas is based mostly B) Giemsa stain
on mitotic count C) Mucicarmine
D) Invade locally but rarely metastasize to distant D) Periodic Acid Schiff
organs ANSWER: C
ANSWER: C
93. Which of the following vulvar lesions is a high-risk cancer
precursor and is associated with HPV?
A) Vulvar intraepithelial neoplasia (VIN)
B) Lichen planus
C) Lichen sclerosus et atrophicus
D) Squamous cell hyperplasia
ANSWER: A
94. Which part of the normal prostatic parenchyma is
composed of glands that may histologically mimic high-
grade prostatic intraepithelial neoplasia?
A) Anterior fibromuscular stroma
B) Peripheral zone
C) Central zone
D) Periurethral tissue
ANSWER: C

95. Why is a nonkeratinizing nasopharyngeal carcinoma


express positivity for Cytokeratin?
A) The cell lineage of the tumor is epithelial and
expresses Cytokeratin filaments
B) The cell lineage of the tumor is hematolymphoid
and expresses Cytokeratin filaments
C) The cell lineage of the tumor is mesenhymal and
expresses Cytokeratin filaments
D) The cell lineage of the tumor is neural and
expresses Cytokeratin filaments
ANSWER: A
PHARMACOLOGY
96. Why should an Ameloblastoma be completely excised?
A) Inadequate excision may lead to local recurrence 1. A 22-year-old woman with binge-eating disorder is seeking
B) Inadequate excision may lead to distant help for her condition. Which of the following drugs might
metastasis be prescribed by her family physician?
C) Inadequate excision will lead to tumor necrosis A) Lisdexamfetamine
D) All of these B) Fluoxetine
ANSWER: A C) Phentermine
D) Liraglutide
ANSWER: A

2. A 24-year-old patient receiving combination therapy for the


treatment of tuberculosis becomes pregnant, although she
has been using oral contraceptives. Which of one the
following drugs is responsible for interfering with the
action of the oral contraceptives, resulting in medication
failure?
A) Ethambutol.
B) Isoniazid.
C) Pyrazinamide.
D) Rifampin.
ANSWER: D

3. A 25-year-old male medical student presents with mild


fever, arthralgia in multiple joints, and hepatomegally.
CBC shows marked eosinophelia, and upon questioning,
the patient reveals a recent volunteer aid trip to Leyte,
about 1 month before presentation. Stool examination and
serum antibody testing are positive for Schistosoma
japonicum . The recommended medication would be:
A) Ivermectin.
B) Chloramphenicol.
C) Praziquantel.
D) Pyrantel pamoate.
ANSWER: C

4. A 25-year-old man comes to your office complaining of


sneezing, coughing, runny nose, and itchy eyes, all
symptoms of allergic rhinitis. He also notes that he has
been tired lately and has been self-medicating with an over-
the-counter antihistamine. Which agent is most likely the
cause of his fatigue?
A) Loratadine D) Stroke.
B) Diphenhydramine ANSWER: B
C) Ketotifen
D) Fexofenadine 11. A 32-year-old woman with schizophrenia responded very
ANSWER: B well to the typical antipsychotic medication prescribed to
control her hallucinations and delusions, but she was very
5. A 25-year-old woman went to her gynecologist to request bothered by the dry mouth she was experiencing. This
an oral contraceptive. Which of the following conditions is effect may be attributed to which action?
an absolute contraindication for the use of these drugs? A) Antiadrenergic
A) Concurrent use of ampicillin. B) Anticholinergic
B) Diabetes mellitus type 1. C) Antidopaminergic
C) Medical history of a deep venous thrombosis. D) Antihistaminergic
D) Recent abortion using mifepristone. ANSWER: B
ANSWER: C
12. A 34-year-old man was brought into the emergency
6. A 27-year-old man with a history of severe shellfish department by the police, who rescued him from the ledge
allergies is brought to the emergency department after of a five-story building. The man was totally despondent
eating a fish stew that contained shrimp. Immediately after and had no interest in continuing to live. He was
entering the emergency department, the patient starts to administered a medication intravenously, and within 2
have difficulty breathing. His pulse becomes weak, and he hours, the darkness seemed to lift, and he indicated an
loses consciousness. Which of the following is the most interest in reuniting with his family. Which of the following
appropriate medication to administer at this time? medications was administered?
A) Albuterol. A) Fluoxetine
B) Epinephrine. B) Selegiline
C) High-dose dopamine. C) Imipramine
D) Isoproterenol. D) Ketamine
ANSWER: B ANSWER: D

7. A 30-year-old hypertensive patient whose blood pressure 13. A 36-year-old man with schizophrenia was prescribed
has been adequately controlled with an angiotensin- haloperidol 5 years ago and was responding very well.
receptor–blocking (ARB) agent just found out that she was Recently, he developed involuntary facial movements that
pregnant. As a consequence, which of the following agents could only be alleviated by increasing his dose of the
would be the best choice for an antihypertensive agent for a medication. This adverse reaction was likely due to which
woman in this situation? of the following mechanisms?
A) β-Adrenergic receptor blockers A) 5-HT receptor sensitization
B) α-Methyldopa B) DA receptor sensitization
C) Monoxidine C) Decreased synthesis of 5-HT receptors
D) Clonidine D) Decreased synthesis of DA receptors
ANSWER: B ANSWER: B

8. A 32-year-old homeless woman was found unresponsive 14. A 39-year-old obese man with type 2 diabetes mellitus is
with a needle in her arm. The response team quickly taking a drug that increases insulin secretion by blocking
administered a dose of naloxone to: ATP-sensitive potassium channels on pancreatic β cells.
A) Desensitize opioid receptors. This drug is:
B) Antagonize the ability of opioids to cross the A) Sitagliptin.
blood-brain barrier. B) Pioglitazone.
C) Competitively inhibit opioid-induced respiratory C) Metformin.
depression. D) Repaglinide.
D) Stimulate the hepatic metabolism of the opioids. ANSWER: D
ANSWER: C
15. A 42-year-old obese man with hyperglycemia seeks his
9. A 32-year-old hypertensive woman just found out she was physician for help. A drug that is approved for both
pregnant. Which one of the following antihypertensive diabetes and obesity that may prove to be useful for this
drugs is contraindicated in pregnant hypertensive women? patient is:
A) Diltiazem. A) Amphetamine
B) Nebivolol. B) Lorcaserin
C) Enalapril. C) Liraglutide
D) Hydrochlorothiazide. D) Phentermine/topiramate
ANSWER: C ANSWER: C

10. A 32-year-old woman was given a prescription for a 16. A 48-year-old man with adrenal insufficiency was
combination oral contraceptive containing both an estrogen prescribed a glucocorticoid and instructed to take it every
and a progestin. The combination of these two hormones is other day. What is the primary advantage of this dosing
better than an estrogen-only contraceptive because the schedule?
addition of the progestin reduces which adverse effect of A) It reduces adverse effects.
estrogen-only administration? B) It is advantageous to achieve elevated and
A) Breast cancer. sustained immunosuppression.
B) Endometrial cancer. C) It minimizes effects following abrupt withdrawal.
C) Myocardial infarction. D) It leads to stabilized hormone levels.
ANSWER: A 23. A 65-year-old man with type 2 diabetes mellitus is taking a
drug that improves the tissue sensitivity of insulin by acting
17. A 55-year-old menopausal woman was recently diagnosed on the peroxisome proliferator–activated receptor. Which
with type 2 diabetes based on her fasting blood glucose of the following is the patient taking?
values. Her HbA1c was <7%, indicating that diet, exercise, A) Acarbose.
and a single drug (monotherapy) may be sufficient to B) Metformin.
regulate her fasting glucose levels. Which of the following C) Repaglinide.
drugs should be prescribed for this patient? D) Pioglitazone.
A) Acarbose. ANSWER: D
B) Colesevelam.
C) Glyburide. 24. A 7-year-old boy is having major difficulty in school with
D) Metformin. inattentiveness and hyperactivity. He is suspected to have
ANSWER: D ADHD. Which neurotransmitter systems in the brain may
be functioning suboptimally?
18. A 56-year-old male complains of mild, intermittent chest A) Dopamine and serotonin
pain and shortness of breath that is exacerbated by exercise. B) Dopamine and norepinephrine
After a thorough cardiac workup, you decide to start the C) Norepinephrine and serotonin
patient on nitroglycerin to relieve these symptoms. Which D) Dopamine and glutamate
of the following best explains the mechanism of action of ANSWER: B
nitroglycerin?
A) It activates phospholipase C to increase 25. A 7-year-old male patient presents complaining of pruritis
intracellular calcium. in the anal area. He is diagnosed as having a pinworm
B) It produces reactive nitrogen species (ONO − ) infection ( Enterobius vermicularis ) and prescribed an
that activate adenylyl cyclase. agent that binds to β-tubulin, preventing polymerization,
C) It produces nitric oxide (NO) that activates thus decreasing ATP level and killing the worm. This drug
guanylyl cyclase. is most likely:
D) It inhibits of calcium influx through L-type A) Albendazole.
calcium channels B) Ivermectin.
ANSWER: C C) Diethylcarbamazine.
D) Pyrantel pamoate.
19. A 6-year-old girl and her mother come to see you because ANSWER: A
the girl's teacher observed episodes of staring and inability
to communicate. These episodes last 3–5 seconds and occur 26. A 72-year-old man with Parkinson disease is being treated
10–20 times during the school day. You are considering with l -DOPA/carbidopa. The beneficial effect of carbidopa
absence seizures. Which antiepileptic medication would may be attributed to which of the following actions?
you try first in this young girl? A) Its conversion to l -DOPA
A) Phenytoin B) Its conversion to dopamine
B) Clonazepam C) Its inability to cross the blood-brain barrier
C) Primidone D) Inhibition of COMT
D) Ethosuximide E) Inhibition of MAO
ANSWER: D ANSWER: C

20. A 62-year-old man with gout experienced a flare-up. Which 27. A 78-year-old man presents for evaluation of a painful rash.
drug did he take to alleviate the acute pain and swelling he He reports a sharp, burning pain radiating from his midback
experienced? to his left side. He noticed a “rash” that spread “like a line”
A) Allopurinol in the same area where he had pain. The rash has a
B) Febuxostat dermatomal distribution from his spine around the left flank
C) Colchicine to the midline of the abdomen. It has erythematous patches
D) Probenecid with clusters of vesicles. He reports a history of chickenpox
ANSWER: C as a child. Among the following, which would be the most
appropriate agent to prescribe?
21. A 62-year-old woman taking medications for ischemic A) Abacavir.
heart disease develops gingival hyperplasia. Which of the B) Entecavir.
following drugs is the most likely culprit? C) Famciclovir.
A) Nitroglycerin D) Oseltamivir.
B) Ranolazine ANSWER: C
C) Metoprolol
D) Verapamil 28. A last resort treatment for P. aeruginosa resistant to all
ANSWER: D other treatment options is the:
A) Penicillins.
22. A 64-year-old woman had a mole on her back that required B) Carbapenems.
dermatological removal using a local anesthetic. These C) Polymyxins.
drugs exert their therapeutic effects primarily by: D) Monobactams.
A) Stimulating activity–dependent Na + channels. ANSWER: C
B) Blocking activity–dependent Na + channels.
C) Stimulating activity–independent Na + channels. 29. A patient asks how nitroglycerin works to relieve anginal
D) Blocking activity–independent Na + channels. pain. The correct answer would be that nitroglycerin:
ANSWER: B A) Constricts coronary arteries to increase blood
flow to the heart.
B)
Increases the oxygen demand in the cardiac ANSWER: C
muscle.
C) Increases ventricular filling to improve cardiac 36. Although the benzodiazepines and the benzodiazepine
output. receptor agonists improve sleep latency, these agents are
D) Promotes vasodilation, which reduces preload not ideal for the treatment of chronic insomnia because
and oxygen demand. they:
ANSWER: D A) Cause CNS depression
B) Have the potential for dependence
30. A patient is admitted to the coronary intensive care unit in C) Lead to daytime sedation
acute decompensated congestive heart failure. The patient D) All of the above
is treated with intravenous furosemide to reduce the level of ANSWER: D
edematous fluid. In addition, an infusion of dobutamine is
given to increase the inotropic state of the heart. At which 37. Although β-lactam antibiotics are considered primarily to
one of the following receptors does dobutamine act to be bactericidal, in some bacteria, they are only
produce this positive inotropic action? bacteriostatic. Which of the following statements best
A) α 1 -Adrenergic. explains this phenomenon?
B) β 1 -Adrenergic. A) The absence of autolysins.
C) β 2 -Adrenergic. B) The absence of porin proteins.
D) Dopamine 1 . C) The formation of filamentous structures.
ANSWER: B D) Increased layers of peptidoglycan in the cell wall.
ANSWER: A
31. A patient is newly diagnosed with active tuberculosis.
Which one of the following drug combinations should be 38. An 18-year-old college student took a pill she bought from
initiated in this patient? a friend and began hallucinating. Which neurotransmitter
A) Amikacin, isoniazid, pyrazinamide, streptomycin. system is thought to underlie the manifestation of
B) Ciprofloxacin, cycloserine, isoniazid, hallucinations?
ethionamide. A) Dopamine
C) Ethambutol, isoniazid, rifabutin, moxifloxacin. B) GABA
D) Isoniazid, rifampin, pyrazinamide, ethambutol. C) Norepinephrine
ANSWER: D D) Serotonin
ANSWER: D
32. A patient with hemophilia A needs a small procedure that
might cause some bleeding. A drug that could be given to 39. An 82-year-old woman with Parkinson disease has been
prevent bleeding problems in this patient is: taking l -DOPA/carbidopa for 7 years. She has recently
A) Tolvaptan. noticed that some abnormal choreiform-like movements
B) Lanreotide. have developed in her arms and legs. These movements are
C) Somatostatin. a typical example of which of the following?
D) Desmopressin. A) l -DOPA-induced psychosis
ANSWER: D B) l -DOPA-induced dyskinesia
C) “On-off” effects of l -DOPA
33. A person who has been taking one drug chronically and D) Toxicity of l -DOPA
experiences a withdrawal syndrome upon discontinuation ANSWER: B
finds relief from these symptoms by taking a different drug.
This is an example of: 40. An agricultural worker is accidentally sprayed with an
A) Craving organophosphate insecticide and is brought to the local
B) Psychological dependence emergency department. He complains of tightness in the
C) Cross-dependence chest and difficulty with vision and was observed to have
D) Tolerance pinpoint pupils and to be profusely salivating. Assuming he
ANSWER: C has been exposed to a cholinesterase inhibitor, the most
appropriate medication for treating his condition would be:
34. A woman who just landed her first job in a big marketing A) Atropine
firm started to deliver a major presentation in the B) Physostigmine
boardroom. Suddenly her hands began to sweat, and she C) Edrophonium
felt her heart beat so fast that she began to feel lightheaded. D) Pilocarpine
Prescribing which of the following medications would be ANSWER: A
the best choice to alleviate this patient's symptoms before
she gives her presentations? 41. An antibiotic is administered once daily in the intensive
A) A benzodiazepine care unit to treat sepsis caused by an abdominal wound.
B) A β-adrenergic receptor blocker Serum and urine concentrations of the drug are monitored
C) A monoamine oxidase A inhibitor during the course of therapy. Ten days after therapy is
D) A serotonin selective reuptake inhibitor discontinued, the drug is still detectable in the urine. Which
ANSWER: B of the following antibiotics was administered?
A) Azithromycin.
35. Activation of the parasympathetic nervous system results in B) Chloramphenicol.
which of the following responses? C) Doxycycline.
A) An increase in heart rate. D) Gentamicin.
B) Vasoconstriction. ANSWER: D
C) Bronchoconstriction.
D) Relaxation of the GI tract.
42. An overly curious ethnobotanist ingested some plant C)
Specifically inhibits hypoxanthine-guanine
material and developed signs of intoxication. Upon delivery phosphoribosyltransferase activity in T cells.
to a hospital emergency department, the patient presents D) Specifically inhibits production of cytokines by T
with bizarre behavior and delirium, facial flushing, high lymphocytes.
heart rate, distended abdomen and full bladder, high fever, ANSWER: D
dilated pupils, dry mouth, clear lungs, and absence of
bowel sounds. Which of the following alkaloids is the most 49. Dantrolene is an antispasticity drug often used for the
likely cause of these symptoms? treatment of spasticity associated with cerebral palsy or
A) Physostigmine. multiple sclerosis. Dantrolene relaxes skeletal muscle by:
B) Reserpine. A) Stimulating GABA receptors on spinal motor
C) Atropine. neurons
D) Pilocarpine. B) Inhibiting excitatory amino acid release
ANSWER: C C) Inhibiting calcium release from the sarcoplasmic
reticulum
43. Antibiotic options for treating MRSA infections include all D) Blocking glutamate NMDA receptors
of the following except: ANSWER: C
A) Vancomycin.
B) Clindamycin. 50. Drugs A, B, C, and D have lipid:water partition coefficients
C) Daptomycin. of 0.1, 1.0, 10, and 100, respectively. Which of these drugs
D) Polymyxins. will cross membranes the fastest?
ANSWER: D A) A
B) B
44. Antibiotics are commonly administered before surgical C) C
procedures. Which of the following statements best D) D
describes the guidelines for perioperative antimicrobial ANSWER: D
prophylaxis?
A) Administer the antibiotic just prior to the 51. Drugs A, B, C, D, and E have therapeutic indices of , 5, 15,
procedure. 20, and 50, respectively. Based on this, which drug has the
B) Begin antibiotic prophylaxis at least 24 hours lowest potential to have adverse or toxic effects?
before surgery. A) A
C) Include an antifungal in the regimen. B) B
D) Select the broadest spectrum antibiotic for C) C
complete coverage. D) D
ANSWER: A ANSWER: D

45. Aspirin can: 52. Generalized tonic-clonic seizures are characterized by a


A) Prevent the formation of TXA 2 . sustained depolarization of cortical neurons with high-
B) Prolong whole blood clotting time. frequency repetitive action potential firing. An antiseizure
C) Shorten bleeding time. drug that acts by which of the following mechanisms is best
D) Inhibit fibrinolysis. suited to treat such seizures?
ANSWER: A A) A voltage-gated sodium-channel blocker
B) A T-type calcium-channel blocker
46. Binding of a drug to its target most often: C) A GABA A receptor positive modulator
A) Involves covalent binding between the target and D) A GABA transporter inhibitor
the drug. ANSWER: A
B) Involves more than one type of weak bond
between the drug and its target. 53. Heparin:
C) Requires long-lasting stable bonds between the A)Has thrombolytic activity.
drug and its target. B)Has most prolonged activity when given orally.
D) Has a similar affinity for the several C)Acts by binding to antithrombin.
stereoisomers of the drug. D)Inhibits the aggregation of platelets caused by
ANSWER: B TXA 2 .
ANSWER: C
47. Corticosteroids are utilized extensively in the treatment of
autoimmune disease and in the prevention of graft rejection 54. How does naproxen control the pain of an inflamed
because they: temporomandibular joint?
A) Alkylate DNA and inhibit the proliferation of B A) It inhibits the activation of sodium channels and
cells. prevents abnormal signal transmission.
B) Are also effective antiinflammatory agents. B) It inhibits COX and prevents the formation of
C) Stimulate cytokine production. prostaglandins.
D) Inhibit purine biosynthesis. C) It stimulates µ receptors and inhibits the release
ANSWER: B of neurotransmitters in the spinal cord.
D) It is an agonist at spinal cord prostanoid
48. Cyclosporine is considered one of the more selective receptors.
immunosuppressive agents because it: ANSWER: B
A) Alkylates DNA and inhibits B-cell proliferation.
B) Specifically inactivates lymphocytes by binding 55. Initial treatment regimens for uncomplicated gonococcal
to CD3 surface antigens on T cells. infections include:
A) Vancomycin.
B) Azithromycin or doxycycline. B) Enhance GABA inhibition
C) Cefotaxime. C) Inhibit glutamate excitotoxicity
D) Ceftriaxone. D) Block calcium channels
ANSWER: D ANSWER: A

56. Mary was prone to developing migraines and wanted to 63. The ability of the cannabinoids to alleviate pain and
take a drug to prevent their occurrence. A drug used for the inflammation may be attributed to activation of:
prevention of severe migraines whose mechanism of action A) CB 1 and CB 2 receptors
involves inhibition of vesicular Acetylcholine release is: B) G-protein–coupled receptors
A) Verapamil C) Transient receptor potential channels
B) Valproate D) All of the above
C) Amitriptyline ANSWER: D
D) Botulinum toxin
ANSWER: D 64. The affinity constant of a drug for its target (K D ) is:
A) An intrinsic property of the binding site of the
57. Methicillin-resistant Staphylococcus aureus (MRSA) is a target molecule for the drug and the drug itself.
common nosocomial pathogen that is increasing in B) The ratio of the reverse to forward rate constants
frequency in community settings. Which of the following for the drug-target binding equation.
statements best describes the most common mechanism of C) Determined by the rate of diffusion of the drug in
resistance by S. aureus ? plasma.
A) Acquisition of the novel protein PBP2a. D) Characterized by A and B above.
B) Increased cell wall repair. ANSWER: D
C) Increased efflux of β-lactams.
D) Reduced permeability to β-lactams. 65. The corticosteroid with high antiinflammatory activity and
ANSWER: A minimal effects on blood pressure after acute
administration is:
58. Metoprolol would be most effective in blocking the ability A) Cortisone.
of epinephrine to: B) Hydrocortisone.
A) Reduce insulin secretion from the pancreas. C) Dexamethasone.
B) Increase the release of renin from the D) Prednisone.
juxtaglomerular apparatus. ANSWER: C
C) Increase glucagon secretion from the pancreas.
D) Dilate the pupil, leading to mydriasis. 66. The loop diuretics have their principal diuretic effect on
ANSWER: B the:
A) Ascending limb of the loop of Henle.
59. Most drugs used for the treatment of Alzheimer disease are B) Distal convoluted tubule.
directed toward increasing the amount of which C) Proximal convoluted tubule.
neurotransmitter in the brain? D) Distal pars recta
A) Norepinephrine ANSWER: A
B) Dopamine
C) Serotonin 67. The most common side effect of oral iron preparations
D) Acetylcholine involves:
ANSWER: D A) The hematopoietic system.
B) The pulmonary system.
60. A 24-year-old motorcyclist had an accident a week ago that C) The gastrointestinal system.
resulted in an open wound on his knee that became D) The kidneys.
infected. When he decided to seek medical help, the ANSWER: C
physician told him that he could not use lidocaine as a local
anesthetic prior to suturing the wound because: 68. The NSAIDs are used on a chronic basis by many who
A) The wound was too old. suffer headache and inflammation. Many individuals do not
B) Lidocaine is a strong acid. realize that the chronic use of these compounds can lead to
C) Lidocaine would be destroyed by plasma major adverse effects on which organ system(s)?
esterases. A) Cardiovascular.
D) Lidocaine would be in a charged form in the B) Hepatic.
infected area and would not readily penetrate the C) Renal.
cell membrane. D) Gastrointestinal.
ANSWER: D ANSWER: D

61. Spironolactone: 69. The primary difference between the first- and second-
A) Competes for aldosterone receptors generation H 1 -antihistamines is their:
B) Inhibits the excretion of K+ A) Ability to enter the CNS.
C) Acts at the late distal tubule B) Efficacy at H 1 receptors.
D) Is characterized by all of the above C) Metabolism by cytochrome P450s.
ANSWER: D D) Efficacy at H 2 receptors.
ANSWER: A
62. The ability of drugs to relieve the pain associated with
migraine headaches is attributed to the ability of these 70. The primary mechanism of action of amphetamines that is
compounds to: beneficial in the treatment of ADHD in children is:
A) Activate 5-HT receptors A) Selective agonism of α 2 -adrenergic receptors.
B)
Selective inhibition of Norepinephrine reuptake. B) Precipitation of a stroke.
C)
Release of Dopamine and Norepinephrine from C) Renal insufficiency.
presynaptic nerve terminals. D) Osteoporosis.
D) Selective inhibition of vesicular Dopamine ANSWER: D
transport.
ANSWER: C 79. When assessing a patient who has been taking a loop
diuretic on a long-term basis, which potential side effect is
71. The primary objective of chemotherapy for a given patient MOST important?
may involve: A) Disturbed sensory perception, visual.
A) Reducing tumor burden. B) Disturbed sensory perception, auditory.
B) Adjunctive therapy following surgery. C) Impaired physical mobility.
C) Complete remission D) Altered mucous membranes.
D) All of the above. ANSWER: B
ANSWER: D
80. When using combination antibiotic therapy, it is important
72. The sympathetic nervous system is characterized by which to administer drugs that work synergistically if possible.
one of the following? Which one of the following represents a combination with
A) Ganglia are located close to the organ innervated. known synergism?
B) The neurotransmitter at postganglionic A) A β-lactam and an aminoglycoside.
sympathetic neuroeffector junctions is B) A penicillin and a cephalosporin.
Norepinephrine. C) Two drugs in which the second drug will displace
C) Postganglionic sympathetic neurons are short. the first from plasma protein–binding sites.
D) The absence of nicotinic cholinergic receptors at D) Two drugs that are eliminated by different routes.
the paravertebral ganglia. ANSWER: A
ANSWER: B
81. Which of the following agents is not absorbed into the
73. The time it takes for the body to decrease the levels of drug systemic circulation and has its actions locally in the
in the circulation by half is known as: gastrointestinal tract?
A) Clearance. A) Amitriptyline
B) Bioavailability. B) Dronabinol
C) Time of distribution. C) Phentermine
D) Half-life. D) Orlistat
ANSWER: D ANSWER: D

74. There is conclusive evidence-based support for the use of 82. Which of the following classes of drugs acts by binding
cannabinoids for the treatment of which condition? reversibly to the 50S ribosomal unit, blocks peptidyl
A) Parkinson disease. transferase, and results in preventing translocation from the
B) Traumatic brain injury. aminoacyl site to the peptidyl site?
C) Cancer. A) Aminoglycosides.
D) Chronic pain in adults. B) Glycylcyclines.
ANSWER: D C) Streptogramins.
D) Tetracyclines.
75. Warfarin: ANSWER: C
A) Acts rapidly when given orally.
B) Is potentiated by barbiturates. 83. Which of the following drugs is indicated for the treatment
C) Is antagonized by protamine sulfate. of either HIV or hepatitis B?
D) Affects the activity of clotting factors. A) Tenofovir.
ANSWER: D B) Etravirine.
C) Abacavir.
76. What characteristics increase the likelihood that a drug will D) Ritonavir.
penetrate the blood-brain barrier and enter the CNS? ANSWER: A
A) Negative charge.
B) High degree of lipophilicity. 84. Which of the following drugs or drug combinations would
C) High molecular weight. you prescribe to treat the erythrocytic stage of malaria?
D) Positive charge. A) Primaquine.
ANSWER: B B) Chloroquine.
C) Atovaquone/proguanil.
77. What is the best initial treatment for a 3-year-old girl D) All of the above
experiencing generalized tonic-clonic seizures daily? ANSWER: B
A) Brain surgery to remove the focus of her seizures
B) Monotherapy with primidone 85. Which of the following is an agonist at cannabinoid type 1
C) Treatment with carbamazepine receptors in both the brain and gastrointestinal tract?
D) Treatment with phenytoin A) Benzphetamine
ANSWER: C B) Lorcaserin
C) Dronabinol
78. What is the major concern with a 72-year-old woman using D) Phentermine/topiramate
prednisone for knee problems for a prolonged period of ANSWER: C
time?
A) Hypertension.
86. Which of the following is true concerning the synthesis and A) Norepinephrine is the neurotransmitter at
storage of biogenic amine neurotransmitters? parasympathetic ganglia.
A) They are stored in and released from vesicles in B) Acetylcholine is the neurotransmitter at
nerve terminals. autonomic ganglia.
B) They are synthesized in perikarya. C) Postganglionic neurons are long and myelinated.
C) Their concentration in the presynaptic cytosol is D) Parasympathetic neurons innervating the
greater than in the vesicles. respiratory system mediate bronchodilation.
D) They are passively transported into vesicles. ANSWER: B
ANSWER: A
94. Which one of the following statements best describes the
87. Which of the following neurotransmitters has its actions basis for the hyperkalemia associated with the reduction in
terminated by enzymatic degradation? blood pressure caused by administration of either
A) Acetylcholine propranolol or losartan?
B) Norepinephrine A) Losartan reduces potassium entry into skeletal
C) Dopamine muscle.
D) Serotonin B) Both drugs reduce aldosterone secretion.
ANSWER: A C) Both drugs block the aldosterone receptor.
D) Both drugs block epithelial Na + channels.
88. Which of the following processes are involved in ANSWER: C
intracellular signaling cascades?
A) Tyrosine phosphorylation. 95. Which types of neurons originate primarily in the
B) Receptor association with and stimulation of G substantia nigra and hypothalamus?
proteins. A) Noradrenergic.
C) Formation of second messengers, such as cAMP. B) Serotoninergic.
D) All of the above. C) Dopaminergic.
ANSWER: D D) GABAergic.
ANSWER: C
89. Which of the following represents an adaptive response to
the long-term use of agonists? 96. While traveling overseas, a 35-year-old man and his wife
A) Increased synthesis of receptors. eat pork prepared by a street vendor, which appears slightly
B) Decreased degradation of receptors. undercooked. Within the next few weeks, they experience
C) Decreased density of postsynaptic receptors. vague abdominal discomfort and generalized weakness.
D) Increased density of postsynaptic receptors. They both notice strange things in their bowel movements
ANSWER: C and take a sample to the physician, who identifies them as
proglottids (tapeworm). Which of the following is the best
90. Which of the following statements regarding drugs' action treatment for these patients?
and cell surface receptors is not correct? A) Albendazole.
A) By acting on receptors, drugs can enhance, B) Mefloquine.
diminish, or block generation or transmission of C) Mebendazole.
signals. D) Praziquantel.
B) The equilibrium dissociation constant (KD) of ANSWER: D
drug binding to receptors can vary widely.
C) Agonist drugs are highly specific for each 97. Why does ethanol, a CNS depressant, cause an initial phase
subtype of receptor in various classes of of excitation following ingestion?
receptors. A) It activates excitatory glutamate receptors.
D) More than one drug molecule may be required to B) It inhibits GABAergic inhibition.
bind to a receptor and elicit a response. C) It reduces activity of a tonically active inhibitory
ANSWER: C system.
D) It blocks serotonin reuptake.
91. Which one of the following agents exerts a bactericidal ANSWER: C
action by inhibiting topoisomerases and DNA gyrase of
microorganisms? 98. You have been asked to volunteer for a Phase I clinical trial
A) Ciprofloxacin. for a new antihypertensive medication. Phase I studies are
B) Nitrofurantoin. important to:
C) Polymyxin B. A) Demonstrate the potential of genetic factors
D) Sulfamethoxazole. contributing to the effects of a new drug in
ANSWER: A humans.
B) Determine the efficacy of a new drug in a specific
92. Which one of the following autonomic receptors are ligand- disease or disorder.
gated ion channels? C) Determine the safety and tolerability of a range of
A) Nicotinic. doses for a new drug in humans.
B) Muscarinic. D) Indicate the effectiveness of a new drug in an
C) α 1 -Adrenergic receptor. animal model of the disease.
D) α 2 -Adrenergic receptor. ANSWER: C
ANSWER: A

93. Which one of the following is a characteristic of the


parasympathetic nervous system?
MICROBIOLOGY

1. A 26-year-old woman with uncomplicated malaria who was


treated initially with chloro- quine now has relapsed. What
is the reason for a chloroquine-treated case of Plasmodium
vivax relapsing?
A) P. vivax has a significant level of chloro- quine
resistance.
B) P. vivax has a persistent erythrocytic stage.
C) P. vivax has a persistent exoerythrocytic stage
(hypnozoite).
D) Chloroquine is not one of the drugs of choice.
ANSWER: C

2. A 40-year-old woman who has suffered from multiple


urinary tract infections (UTIs) is found to have a stone in
her left kidney. Which agent of UTI is particularly
associated with stone formation?
A) Proteus mirabilis
B) Morganella morgana
C) Cronobacter sakazakii
D) Enterococcus faecalis
ANSWER: A

3. A 60-year-old man develops acute bacterial meningitis. A


Gram-stained film of the CSF on admission shows
pleomorphic Gram-negative coccobacilli. Which of the
following antimicrobials should be administered as initial
therapy?
A) A rifamycin (e.g., rifampicin)
B) An aminoglycoside (e.g., gentamicin)
C) An extended-spectrum penicillin (e.g., ampicillin)
D) A third-generation cephalosporin (e.g.,
ceftriaxone)
ANSWER: D

4. A neutropenic patient presents with sepsis and an acute


kidney injury. The patient is resuscitated and treated
empirically with piperacillin/tazobactam. On review,
infection associated with a long intravascular line is
suspected and blood cultures yield Staphylococcus aureus,
which is subsequently shown to be meticillin resistant.
What antimicrobial agent would be appropriate?
A) Flucloxacillin
B) Vancomycin
C) Meropenem
D) Doxycycline
ANSWER: B
5. A nonnucleoside analog that inhibits herpesvirus DNA B)
Gonococci cause disease by infecting mucosal
replication is surfaces and vaccines are ineffective at
A) Acyclovir preventing these types of infections
B) Amantadine C) The gonococcal capsule is identical to a human
C) Cytarabine antigen and is poorly antigenic in humans
D) Foscarnet D) Gonococci lack a capsule and exhibit high levels
ANSWER: D of antigenic variation making it difficult to
develop an effective vaccine
6. All Paramyxovirus infections are associated with ANSWER: D
neurological complications, EXCEPT:
A) Nipah virus 13. Cells transformed by viruses share which ONE of the
B) Mumps following properties?
C) Respiratory syncytial virus A) A full range of viral genes are usually expressed
D) Measles B) Cells are usually particularly sensitive to
ANSWER: C apoptosis
C) Expressed viral genes often modulate cell cycle
7. An important aspect of antiviral responses is cell-mediated D) Interfering with the virus reverses the
immunity. Which of these cell types is important in this transformed phenotype
process? ANSWER: C
A) T helper 17 cells expressing IL-17 and IL-22
B) T regulator cells expressing IL-10 14. Corynebacterium diphtheriae is:
C) T helper 1 cells expressing IFNγ A) Mainly responsible for meningitis
D) T helper 2 cells expressing IL-4, IL-5 and IL-13 B) Transmitted from animal to person by
ANSWER: C consumption of dairy products
C) Transmitted from person to person through
8. Antimicrobial drug resistance genes: respiratory droplets
A) Are genes present in all bacteria D) Mainly present in north Europe
B) Are often spread by horizontal gene transfer ANSWER: C
C) Are the genes responsible for plasmid replication
D) Can transfer to humans, rendering humans 15. Dane particles are associated with
resistance to drugs A) Hepatitis A virus
ANSWER: B B) Hepatitis B virus
C) Hepatitis C virus
9. As a chronic infection, the symptoms of tuberculosis may D) Hepatitis E virus
be difficult to distinguish from other chronic conditions. A ANSWER: B
patient from a Southeast Asian country who has a cough
and is a heavy smoker and who has lost a significant 16. Detection of Mycoplasma pneumoniae infection can be
amount of weight is reviewed. His chest X-ray does not undertaken using differing methodologies. In detecting
show clear signs of TB. He has a BCG scar. In this case, a infection which of the following methods would show the
positive IGRA TB test would indicate which of the most useful information in detecting early infection with
following? this pathogen?
A) That he has been exposed to TB but does not A) Culture
show that he has active disease B) Molecular detection (e.g., PCR)
B) That TB is the most likely diagnosis C) ELISA
C) That he still has active immunity from his BCG D) Clinical signs (e.g., cough, tracheobronchitis)
vaccine ANSWER: B
D) That he may have had nontuberculous
mycobacterial infection. 17. Disinfection of day care center play tables with 70%
ANSWER: A ethanol is least likely to affect the viability of
A) Cytomegalovirus
10. Bacteria lacking superoxide dismutase are B) Parainfluenza virus
A) Heterotrophs C) Respiratory syncytial virus
B) Obligate anaerobes D) Rotavirus
C) Aerobes ANSWER: D
D) Facultative anaerobes
ANSWER: B 18. Early antibiotic intervention for anthrax can, if given
sufficiently early, result in a favourable outcome for the
11. Bacterial genomes: patient. Vaccines can be administered to prevent disease
A) Contain introns occurrence and, in the case of anthrax vaccines, they can be
B) Are characterised by overlapping genes used as part of a postexposure treatment plan. Which of the
C) Are generally circular following is a current licensed anthrax vaccine for humans?
D) Are always linear A) Live, heat-attenuated vaccine
ANSWER: C B) Poly- d -glutamic acid vaccine
C) Recombinant protein vaccine
12. Capsule-based vaccines have been highly effective against D) Culture supernatant vaccine
meningococci. Why are there no currently available ANSWER: D
vaccines for gonococcal infections?
A) Vaccines are not required for gonococcal 19. Effective vaccines or antiviral drug treatments are available
infections as antibiotics are highly effective for the following infections, EXCEPT:
A) Respiratory syncytial virus 25. Hepatitis D virus (HDV) is a defective virus dependent on
B) Parainfluenza virus type 1 hepatitis B virus (HBV) for replication.What is the essential
C) Mumps element that HDV requires from HBV?
D) Measles A) Hepatitis B core antigen (HBcAg)
ANSWER: B B) Hepatitis B virus DNA (HBV DNA)
C) Hepatitis B e antigen (HBeAg)
20. Enteroviruses cause a variety of different infections. Which D) Hepatitis B surface antigen (HBsAg)
one of the following statements best describes these ANSWER: D
infections?
A) Meningitis can be caused by several different 26. Herpes simplex virus and varicella-zoster virus are
enterovirus types, but poliovirus is the most neurotropic viruses. Which statement best describes them?
common cause together with species B A) Infection is associated with an increased risk of
enteroviruses such as echovirus 30 and CNS tumours
coxsackievirus B5 serotypes B) Vaccines are available against these viruses
B) Hand, food and mouth disease (HFMD) is usually C) Laboratory diagnosis is mostly based on virus
a severe infection that is often caused by species isolation in tissue culture
A enteroviruses such as EV-A71, CAV6 and D) Latency is established in sensory ganglia
CAV16 ANSWER: D
C) Enterovirus D68 (EV-D68) is a rare but important
cause of paralysis in children 27. HIV-1 is classified as a member of the Lentivirus genus in
D) Species C enterovirus infections are common in the Retroviridae family. Lentiviruses:
children A) Contain a DNA genome
ANSWER: C B) Cause tumors in mice
C) Infect cells of the immune system
21. Enteroviruses, rhinoviruses, parechoviruses and hepatitis A D) Have related sequences endogenous in normal
virus are picornaviruses known to infect humans. Which cells
one of the following statements best describes these ANSWER: C
viruses?
A) Enteroviruses, rhinoviruses, parechoviruses and 28. How is Clonorchis sinensis (Chinese liver fluke) most
hepatitis A virus are members of the Enterovirus likely transmitted to humans?
genus A) Fish ingestion
B) Enteroviruses are nonenveloped RNA viruses B) Mosquito bite
C) Rhinoviruses are divided into four species: C) Swimming or water contact
rhinovirus A, B, C and D D) Rare beef ingestion
D) Several serotypes of hepatitis A virus are known ANSWER: A
ANSWER: B
29. Human obligate intracellular pathogens that depend on the
22. Epstein–Barr virus, HHV 6 and HHV 7 are lymphotropic host cell for ATP production are
viruses. Which statement best describes them? A) Bacteriophages
A) Transmission is mostly through saliva B) Mycoplasma species
B) Vaccines are in routine use against these viruses C) Prions
C) The viruses are known tumor-forming agents in D) Rickettsia species
man ANSWER: D
D) Bone marrow suppression follows infection with
the viruses 30. IgE mediate histamine release is classified as what type of
ANSWER: A hypersensitivity reaction?
A) Type I
23. Following an abdominal operation, a patient becomes B) Type II
pyrexial and suffers from right upper quadrant pain. C) Type III
Ultrasound reveals an abscess, and microscopic D) Type IV
examination of a blood culture demonstrates a mixed ANSWER: A
Gram-negative (with rod-shaped bacteria) infection. What
organisms are most likely to be responsible? 31. In a case of tetanus occurring in a homeless person
A) Actinomyces presenting with progressive paralysis, which of the
B) Peptostreptococci following is true?
C) Bacteroides fragilis and coliforms A) Patients do not develop protective antibodies
D) Neisseria after recovery.
ANSWER: B B) Immediate active immunization is essential.
C) There should be evidence of a significant wound
24. Foodborne illnesses due to Clostridia are: entry for infection.
A) Mild and self-limiting D) Isolation of Clostridium tetani is required to
B) Toxin mediated confirm the diagnosis.
C) Preventable by extended maintenance of food at ANSWER: A
boiling point during cooking
D) Highly transmissible from person to person 32. In a patient newly diagnosed with HIV infection, a
ANSWER: B thorough assessment was performed to enable clinical
staging. Which of the following is considered an AIDS-
defining criterion?
A) Persistent generalised lymphadenopathy
B) Herpes zoster B) Presence of foul-smelling pus
C) Oral candidiasis C) Isolation of causative organisms from pus
D) Cytomegalovirus (CMV) retinitis D) Failure to respond to treatment with
ANSWER: D metronidazole
ANSWER: B
33. In a person with HIV infection, potentially infectious fluids
include all of the following except: 41. Pseudomonads and non fermenters are environmental
A) Blood organisms (not parasitic or symbiotic). Control/prevention
B) Saliva visibly contaminated with blood of infections due to these agents in health-care facilities is
C) Urine not visibly contaminated with blood best achieved by:
D) Genital secretions A) Cleaning the clinical environment and equipment
ANSWER: C B) Isolation of all infected patients
C) Careful antibiotic stewardship
34. In the replication cycle of HCV, which one of the following D) Careful antibiotic stewardship and cleaning the
statements is correct? clinical environment and equipment
A) The hepatocyte receptor for HCV is the sodium ANSWER: D
taurocholate cotransorting polypeptide (NTCP).
B) The viral genome enters the cell nucleus prior to 42. Regarding Human Papilloma Virus 16, which of the
replication. following statements is correct?
C) Negative-strand RNA is generated as a A) It causes the majority of genital warts
replicative intermediate during the cycle. B) It is the most oncogenic of all the high-risk HPV
D) The protein derived from the NS5A gene is types
encapsidated within mature virus particles. C) It is infrequently detected in cervical cancer
ANSWER: C D) It is responsible for around 30% of cervical
cancers alone
35. Individuals may suffer from recurrent Str. pyogenes ANSWER: B
infections in spite of a strong immune response to
components of the infecting strain. Which of the following 43. Rotaviruses are a leading cause of acute gastroenteritis in
explains the lack of protective immunity? children. Which of the following statements about
A) The capsule rotaviruses is correct?
B) The C5a peptidase A) They possess a nonsegmented genome
C) Extensive polymorphism of the M protein B) Their genome comprises single-stranded RNA
D) The group A antigen C) They possess a lipid envelope
ANSWER: C D) They evolve through genetic reassortment
ANSWER: D
36. Listeria monocytogenes is:
A) A pathogen specific to humans 44. Smallpox was officially declared to be eradicated from the
B) A member of a Gram-negative genus of bacteria world on 9 December 1979. Which ONE of the following
C) A member of a genus of bacteria only containing factors was the most crucial in ensuring the success of the
Listeria monocytogenes eradication program?
D) A bacterium causing predominantly foodborne A) Effective antiviral drugs were available
disease B) Smallpox had no animal reservoir
ANSWER: D C) Universal vaccination was readily achieved
D) Vaccination gave lifelong immunity
37. Members of the Parvovirus family ANSWER: B
A) Encode a DNA polymerase
B) Only infect vertebrates 45. Sterile instruments are an essential requirement for surgery.
C) Have a single-stranded circular DNA genome Which of the following methods should be used to sterilize
D) Preferentially replicate in dividing cells a retractor used in abdominal surgery?
ANSWER: D A) Ionising radiation
B) Immersion in glutaraldehyde
38. MHC class II molecules are critically important in what C) Dry heat delivered in an oven
immunologic process? D) Moist heat delivered in an autoclave
A) Antigen presentation ANSWER: D
B) Phagocytosis
C) Immunoglobulin class switching 46. Superoxide dismutase-containing bacteria
D) CD8 T cell cytotoxicity A) Need superoxide to grow
ANSWER: A B) Are frequently obligate anaerobes
C) Grow slowly in the presence of CO2
39. Negri bodies are associated with D) Produce hydrogen peroxide from hydrogen ion
A) Cytomegalovirus infections and the superoxide free radical (O2•2)
B) Herpes simplex virus infections ANSWER: D
C) Rabies virus infections
D) Rubella virus infections 47. The cells of the immune system develop from
ANSWER: C haematopoietic progenitor cells and can be classified as
either lymphoid or myeloid derived. Which one of the cell
40. Nonclostridial anaerobic infections often demonstrate types below is lymphoid?
which clinical sign/indicator? A) Monocyte
A) Lack of necrotic tissue B) B cell
C) Macrophage B) IgM
D) Neutrophil C) IgA
ANSWER: B D) IgE
ANSWER: C
48. The diagnosis of rotavirus gastroenteritis requires
laboratory examination of a faecal specimen. Which of the 55. The individual most likely to develop chronic liver disease
following statements regarding rotavirus diagnosis is is a
correct? A) 1-month-old infant infected with HBV
A) Up to 10 11 virions per gram of stool are excreted B) 22-year-old coinfected with HBV and HDV
during symptomatic infection C) 26-year-old alcoholic intravenous drug abuser
B) Molecular methods are less likely to detect infected with HBV
asymptomatic shedding D) 30-year-old infected with yellow fever virus
C) Electron microscopy is a highly sensitive and ANSWER: A
specific diagnostic method
D) Cell culture is a commonly employed diagnostic 56. The interaction of two IgG molecules binding to antigen
method in clinical laboratories followed by the binding of C1 to the Fc portion of the
ANSWER: A antibody results in which of the following?
A) Initiation of antigen presentation
49. The distinction between bacterial groups using the Gram B) Initiation of classic complement pathway
stain remains a vital component of clinical specimen C) Initiation of alternative complement pathway
analysis. Which of the following features can be D) Initiation of the mannose-binding lectin-binding
distinguished by this method? complement pathway
A) The presence of flagella on the cell ANSWER: B
B) Distinction between cocci and bacilli
C) The antibiotic sensitivity of cells 57. The laboratory isolates Ps. aeruginosa from the
D) The presence of a capsule bloodstream of a 40-year-old patient with multiple sclerosis
ANSWER: B (MS). Which of the following tests is most likely to identify
the source of the infection?
50. The following can be used in the treatment of Chlamydia A) Antimicrobial susceptibility tests
psittaci or Chlamydia pneumoniae infection except: B) Strain typing by MLST
A) Doxycycline C) Oxidase test
B) Gentamicin D) Urine culture
C) Clarithromycin ANSWER: D
D) Azithromycin
ANSWER: B 58. The minimal concentration of alcohol necessary to kill
bacteria and enveloped viruses is
51. The following factors influence the progression of liver A) 40%
fibrosis in patients with chronic HCV infection, EXCEPT: B) 50%
A) Level of alcohol intake C) 60%
B) HIV coinfection D) 70%
C) Older age at infection ANSWER: D
D) Plasma viral load
ANSWER: D 59. The most common cause of congenital infections is
A) CMV
52. The following statements are true except: B) HSV
A) Chlamydia trachomatis infections may result in C) Parvovirus
infertility in women. D) Rubella virus
B) Women are usually asymptomatic with C. ANSWER: A
trachomatis infection of the genital tract.
C) All males are symptomatic from C. trachomatis 60. The normal flora of the large intestine consists mainly of
infection of the genital tract. A) Bacteria
D) C. trachomatis urethritis may be treated with B) Fungi
ofloxacin. C) Protozoa
ANSWER: C D) Viruses
ANSWER: A
53. The identification of bacterial isolates from clinical
specimens is a key step in identifying the source and public 61. The properties of bacteria change as they pass through the
health implications of infections. Which of the following cell cycle and adopt different patterns of growth in liquid or
methods applied to isolates can most rapidly assist in on solid media. Which of the following growth states or
achieving a presumptive identity. patterns produces the highest level of clinically significant
A) Gram stain phenotypic antibiotic tolerance?
B) 16S directed PCR A) Growth in colonies on agar
C) Biochemical tests B) Exponential growth in liquid
D) Antibiotic susceptibility tests C) Stationary growth in liquid
ANSWER: A D) Biofilm growth
ANSWER: D
54. The immunoglobulin class most frequently responsible for
inhibition of bacteria on mucosal surfaces is: 62. The typical course of an untreated HIV infection extends
A) IgG over 10 or more years. There is usually a long period
(clinical latency) between the time of primary HIV C) Diarrhea
infection and the development of AIDS. During this period D) Jaundice and hepatitis
of clinical latency: ANSWER: B
A) HIV is not detectable in the plasma
B) CD4 cell counts remain unchanged 69. What is a characteristic of the adaptive immune response
C) Virus replicates at a very slow rate and not of the innate response?
D) Virus is present in lymphoid organs A) Physical barriers
ANSWER: D B) Chemical barriers
C) Clonal expansion of effector cells
63. There are eight known human herpesviruses. Which D) Inflammatory mediators
statement best describes them? ANSWER: C
A) Herpesviruses are enveloped dsRNA viruses
B) Antivirals have limited effect against 70. What is a mass of fungal filaments called?
herpesviruses A) Pseudohyphae
C) Herpesviruses establish permanent infection of B) Hyphae
the host C) Mycelium
D) Most primary herpesvirus infections occur in D) Septum
adulthood ANSWER: C
ANSWER: C
71. What is endotoxin?
64. There is robust evidence to suggest that HR-Human A) A component of the Gram-negative cell surface
Papilloma Virus infection is associated with which group of B) Any diffusible protein secreted into the external
cancers? medium
A) Cervical, lung, bladder, anal and vaginal C) A protein toxin that acts specifically on
B) Vaginal, vulval, cervical, anal, penile and enterocytes within the gut wall
oropharyngeal D) A cytotoxin that inhibits adenylate cyclase and
C) Penile, prostate, bladder, testicular and anal thus interferes with phagocyte function
D) Cervical, breast and ovarian ANSWER: A
ANSWER: B
72. What is the target organ of yellow fever virus?
65. Three poliovirus types are known to cause paralytic polio. A) Lung
Which one of the following statements best describes B) Liver
polio? C) Brain
A) Despite international poliovirus eradication D) Lower intestines
efforts, the number of clinical polio cases has not ANSWER: B
reduced significantly over the past 30 years
B) Globally, poliovirus type 2 has been on the 73. What roundworm is most likely to be transmitted by
increase since 1999 ingestion of food or water contaminated with feces?
C) Approximately 1 in 10 poliovirus infections in A) Ascaris lumbricoides
children leads to paralysis B) Enterobius vermicularis
D) Both live-attenuated and killed-inactivated C) Necator americanus
poliovirus vaccines have been used effectively to D) Taenia saginata
prevent poliovirus infection ANSWER: A
ANSWER: D
74. What roundworm is transmitted by filariform larvae that
66. To treat a patient with a life-threatening fungal infection, are found in the soil and penetrate the skin?
you choose an antifungal drug that causes pore formation in A) Dracunculus medinensis
the fungal membrane and actually kills the cells. Which B) Enterobius vermicularis
drug would this be? C) Strongyloides stercoralis
A) Amphotericin B D) Toxicara canis
B) Griseofulvin ANSWER: C
C) Ketoconazole
D) Nystatin 75. Which antibiotic is most appropriate in the treatment of
ANSWER: A enteric fever caused by S. Typhi?
A) Ampicillin
67. Virulence factors are molecules produced by pathogens B) Gentamicin
(bacteria, viruses, fungi and protozoa). The virulence of B. C) Ciprofloxacin
anthracis is largely dependent on the production of two D) Chloramphenicol
tripartite toxins and which of the following? ANSWER: C
A) Polysaccharide capsule
B) Poly-d-glutamic acid capsule 76. Which immune mechanism is characteristically associated
C) Endotoxin with helminth parasites?
D) Dipicolinic acid A) IgG and neutrophil activation
ANSWER: B B) IgE and eosinophil activation
C) MHC class I and CD8 T cell activation
68. What are the most common symptoms of acute HIV D) IgG and macrophage activation
infection? ANSWER: B
A) Rash and sore throat
B) Fever and malaise
77. Which major antibody molecule has the ability to cross the A) C reactive protein
placenta? B) White cell count
A) IgG C) Widal test
B) IgA D) Blood cultures
C) IgM ANSWER: D
D) IgE
ANSWER: A 86. Which of the following is the major mechanism resulting in
the persistence of chronic hepatitis B?
78. Which of the cytokines attract neutrophils and inhibit A) Evasion of host immune surveillance.
bacteria? B) Integration of viral genome into a host
A) IFN-gamma chromosome.
B) IL-8 C) Presence of genome in the form of covalently
C) IL-2 closed circular DNA.
D) IL-6 D) Viral latency in central nervous system.
ANSWER: B ANSWER: C

79. Which of the following antiviral agents against hepatitis B 87. Which of the following is the most likely interpretation of a
virus is most prone to development of resistance? positive influenza A virus laboratory result in an individual
A) Adefovir with underlying clinical risk factors for severe disease who
B) Entecavir received a single dose of vaccine 6 weeks ago?
C) Lamivudine A) Vaccine has not had time to induce protective
D) Pegylated interferon immunity
ANSWER: C B) Multiple doses of influenza vaccine are necessary
to induce a protective immune response
80. Which of the following circumstances may enhance HIV C) Influenza vaccine has poor efficacy in those with
transmissibility in a person who has unprotected sexual underlying risk factors
intercourse? D) The infection is with a zoonotic influenza virus
A) Discordant host HLA Class I genotype for which vaccine does not provide any
B) Circumcised male status protection
C) Human herpes virus type 6 (HHV6) infection ANSWER: D
D) Primary HIV infection in the sexual partner
ANSWER: D 88. Which of the following is the most resistant to destruction
by chemical and heat?
81. Which of the following components is present in gram A) Spores of Aspergillus fumigatus
negative bacteria but nit in gram positive bacteria? B) Mycobacterium tuberculosis
A) Peptidoglycan C) Ebola virus
B) Lipid A D) Spores of Bacillus anthracis
C) Capsule ANSWER: D
D) Flagella
ANSWER: B 89. Which of the following is the tapeworm acquired from
eating undercooked pork?
82. Which of the following factors is least likely to be A) Trichinella spiralis
associated with long-term carriage of Salmonella ? B) Echinococcus granulosus
A) HIV+ status C) Taenia saginata
B) Gallbladder abnormalities D) Taenia solium
C) Infectious dose ANSWER: D
D) Serotype
ANSWER: C 90. Which of the following protozoans is free living and is such
that acquisition does not generally indicate fecal
83. Which of the following features differentiates fungal cells contamination?
from human cells? A) Acanthamoeba
A) 80S ribosomes B) Dientamoeba fragilis
B) Presence of an endoplasmic reticulum C) Entamoeba histolytica
C) Ergosterol as the major membrane sterol D) Entamoeba coli
D) Enzymes that allow them to use carbon dioxide ANSWER: A
as their sole carbon source
ANSWER: C 91. Which one of the following cell types expresses receptors
for IgE on its cell surface that stimulate the cell to mount a
84. Which of the following features of Neisseria meningitidis is response to parasites such as worms?
the most important virulence factor involved in fulminant A) T cells
sepsis and meningitis? B) B cells
A) The capsule C) NK cells
B) Neisseria exotoxin D) Mast cells
C) The bacterial oxidase enzyme ANSWER: D
D) Outer membrane lipooligosaccharide
ANSWER: D 92. Which one of the following conditions is characteristically
caused by a preformed toxin produced by Staphylococcus
85. Which of the following investigations would be most useful aureus?
in the diagnosis of enteric fever? A) Bloodstream infection
B) Food poisoning D) Modifying cellular RNA polymerase
C) Cellulitis ANSWER: B
D) Urinary tract infection
ANSWER: B

93. Which ONE of the following diseases is NOT associated


with adenovirus infection?
A) Gastroenteritis
B) Pharyngoconjunctival fever
C) Vesicular skin rash
D) Viral pneumonia
ANSWER: C

94. Which one of the following pathogenic yeasts is not a


common member of the normal human flora or microbiota?
A) Candida albicans
B) Malassezia globose
C) Cryptococcus neoformans
D) Candida tropicalis
ANSWER: C

95. Which statement regarding Ebola virus is true?


A) ELISA is the gold standard diagnostic tool
B) An effective vaccine is readily available
C) There is no currently approved treatment
D) Case fatality rate ranges from 10 – 30%
ANSWER: C

96. Which statement regarding fungi is correct?


A) All fungi are able to grow as yeasts and molds.
B) Although fungi are eukaryotes, they lack
mitochondria
C) Fungi are photosynthetic
D) Fungi have one or more nuclei and chromosomes
ANSWER: D

97. Which two cytokines play an important role in attracting


neutrophils to the site of an infection?
A) IFN-alpha and IFN-gamma
B) IL-8 and IL-17
C) IL-2 and IL-4
D) IL-6 and IL-12
ANSWER: B

98. With regard to the determinants of the rate of disease


progression from the point of HIV acquisition, which one
of the following is associated most with rapid progression
to AIDS?
A) HIV viral load set-point 6 months after primary
HIV infection
B) CD4+ cell count 6 months after primary HIV
infection
C) Heterozygosity for a 32-bp deletion in the
chemokine receptor CCR5 (CCR5-Δ32)
D) The presence of HLA B*5701 allele
ANSWER: A

99. Within the innate immune response, this cell acts by killing
virus-infected cells:
A) T cell
B) NK cell
C) Macrophage
D) Neutrophil
ANSWER: B

100. β-lactamases confer antibiotic resistance by


A) Altering antibiotic permeability
B) Modifying antibiotic structure
C) Altering 70S ribosome structure

You might also like